Вы находитесь на странице: 1из 72

2002 Self-Assessment Exercise XV.

. Disorders of the blood and neoplastic disorders [Return to Category List] Questions Question 4. Answer.

A 6-year-old boy who has severe factor VIII deficiency hemophilia develops increasing swelling of the right distal forearm. There is no history of trauma. The peripheral circulation is normal, and there is no joint involvement. Of the following, The MOST important complication of bleeding in this location is: A. B. C. D. E. blood loss muscular damage neurologic impairment tendon shortening vascular damage Answer.

Question 26.

An 11-year-old girl who has chronic renal failure of unknown cause is found to have a hemoglobin of 6.5 g/dL (65 g/L) on an incidental blood count. She does not report increasing fatigue. She is being maintained on chronic ambulatory peritoneal dialysis. Results of other laboratory studies include: reticulocyte count, 0.3% (3 x 10-3); white blood cell count, 7,900/cu mm (7.9 x 109/L); platelet count, 399 x 103/cu mm (399 x 109/L); mean corpuscular volume, 84 fL; and creatinine, 2.5 mg/L (221 mcmol/dL). Of the following, the MOST appropriate therapy for this girl's anemia is: A. B. C. D. E. ferrous sulfate orally ferrous sulfate parenterally multivitamins that include copper recombinant erythropoietin subcutaneously red blood cell transfusion Answer.

Question 47.

A 2-year-old boy presents for evaluation of a chronic pruritic eruption. His medical history is remarkable for recurrent epistaxis, otitis media, and pneumonia. Physical examination

reveals erythematous, slightly scaling patches on the trunk and in the antecubital and popliteal fossae. Petechiae are present profusely. Of the following, these findings are MOST suggestive of: A. B. C. D. E. acrodermatitis enteropathica ataxia telangiectasia atopic dermatitis Langerhans cell histiocytosis Wiskott-Aldrich syndrome Answer.

Question 90.

You are seeing a 9-year-old boy for a health supervision visit. He was treated for acute lymphoblastic leukemia at 4 years of age and has been off therapy and in remission for almost 2 years. Of the following, the MOST important site to examine for potential relapse in this boy is (are): A. B. C. D. E. hips liver lymph nodes testes tonsils

Question 112. Answer. A 19-year-old previously healthy boy has developed a deep vein thrombosis of the right femoral vein. One week ago he sprained his ankle playing football and has been ambulating on crutches since that time. Of the following, the MOST likely cause of his venous thrombosis is: A. B. C. D. E. circulating lupus anticoagulant hereditary thrombophilia ingestion of androgenic steroids the injury and immobility unrecognized compression of the femoral vein

Question 134. Answer.

A 4-year-old girl developed a fever to 104F (40C) and a diffuse, raised purpuric rash this morning. Laboratory studies reveal: white blood cell count, 5,100/cu mm (5.1 x 109/L); hemoglobin, 11.8 g/dL (118 g/L); platelets, 189 x 103/cu mm (189 x 109/L); prothrombin time, 11.8 sec; partial thromboplastin time, 28 sec; and fibrinogen, 250 mg/dL (2.5 g/L). Of the following, the MOST likely etiology of the purpura is: A. B. C. D. E. disseminated intravascular coagulation Henoch-Schnlein purpura hereditary telangiectasia inherited platelet dysfunction meningococcemia

Question 156. Answer. A 15-month-old boy develops fatigue and pallor. Physical examination results are otherwise normal. Laboratory studies reveal: hemoglobin, 2.9 g/dL (29 g/L); white blood cell count, 6,400/cu mm (6.4 x 109/L); differential count, 43% neutrophils, 49% lymphocytes, and 8% monocytes; platelets, 298,000/cu mm (298 x 109/L); mean corpuscular volume, 72 fL; reticulocyte count, 0.1% (1 x 10-3); indirect bilirubin, 0.2 mg/dL (3.42 mcmol/L); and fetal hemoglobin, 2% (normal for age). Of the following, the MOST likely diagnosis is: A. B. C. D. E. anemia of chronic disease Diamond-Blackfan anemia Fanconi anemia iron deficiency anemia transient erythroblastopenia of childhood

Question 178. Answer. In examining a 4-year-old girl who is new to your practice, you discover that she has rudimentary thumbs and is well below the 5th percentile for both weight and height. You also observe irregular hyperpigmentation on the trunk and anogenital areas. Of the following, the MOST likely hematologic disorder associated with these findings is: A. B. C. acute lymphoblastic leukemia Bloom syndrome Diamond-Blackfan anemia

D. E.

Fanconi anemia thrombocytopenia and absent radii (TAR) syndrome

Question 201. Answer. You identify a right-sided abdominal mass in a 2-month-old girl during a health supervision visit. The mass is firm but not tender. Abdominal ultrasonography reveals a solid mass pushing the right kidney downward. Abdominal radiography reveals no calcifications in the mass. Of the following, the MOST likely diagnosis is: A. B. C. D. E. adrenocortical carcinoma hepatoblastoma neuroblastoma rhabdomyosarcoma Wilms tumor

Question 223. Answer. A 7-year-old boy presents with bruising, pallor, intermittent "achy" pain in the extremities, and intermittent low-grade fever of 2 weeks' duration. Findings on laboratory studies include: hemoglobin, 5.5 g/dL (55 g/L); white blood cell count, 2,100/cu mm (2.1 x 109/L); mean corpuscular volume, 76 fL; lactate dehydrogenase, 230 IU/L; and uric acid, 5.5 mg/dL (327 mcmol/L). Of the following, the MOST likely diagnosis is: A. B. C. D. E. acute lymphoblastic leukemia aplastic anemia Fanconi anemia infectious mononucleosis parvovirus infection

Question 245. Answer. You diagnose severe factor VIII deficiency hemophilia in a 3-year-old boy who has no family history of hemophilia. The parents want to understand the disorder better. Of the following, a TRUE statement is that: A. B. family history is negative in 30% of cases identification of female carriers is difficult

C. D. E.

it is a sex-linked recessive disorder with variable penetrance moderate hemophilia (factor VIII levels of 1% to 5%) is most common prenatal diagnosis is possible in only 25% of families

Question 264. Answer. A medical student asks you the difference between prothrombin time (PT) and partial thromboplastin time (PTT). Which of the following statements is TRUE?: A. B. C. D. E. Only the PTT is prolonged by heparin Only the PTT is prolonged in hemophilia and von Willebrand disease The International Normalized Ratio is derived from the PTT The PTT is preferred for monitoring coumarin dosage The PTT measures all the components of the PT except factor V

Answers Critique 4. Preferred Response: C

[View Question] Children who have hemophilia can develop severe peripheral neurologic deficits when hematomas compress nerves. Because such hematomas usually do not involve the joint, they may be assessed as less serious soft-tissue hematomas. Most smaller soft-tissue hematomas in less critical sites do not require treatment, but compression within closed spaces, such as the forearm (Figure 4A), can result in compartment syndrome that can compromise nerve function and blood flow. Vascular compromise is usually obvious, but peripheral nerve involvement may be overlooked. These neurologic deficits often are slow to resolve and may result in permanent deficit. In older children, increasing pain out of proportion to the size of the hematoma, numbness, and paresis are critical signs. Frequently weakness is mistaken for intentional disuse because of pain. This is even more likely to occur in younger children who are less verbal. As with many bleeding episodes in children who have hemophilia, there often is no history of trauma. A delay of hours to days from the time of injury to the development of a clinically evident hematoma may result in the child forgetting the trauma. In addition, many affected children develop hematomas after injuries that would have been considered trivial. Such episodes are more likely to occur in those who have severe hemophilia, but they are seen in children who have moderate and even mild disease.

Significant blood loss can complicate hematoma formation, but this usually occurs in the thigh or retroperitoneum where substantial blood loss may not be readily apparent on physical examination. Iatrogenic hematomas can be serious. Routine venipuncture, when performed appropriately, is without danger, but it should be followed by at least 5 minutes of firm finger pressure. In contrast, femoral or jugular venipuncture and any arterial puncture should not be undertaken without prior factor replacement. Otherwise, serious hematomas can result. In the jugular area, this can lead to compression of vital structures, as exemplified by vocal cord hematomas and subsequent asphyxia resulting from local anesthetic blocks during dental work. In the femoral triangle, large hematomas may form before swelling is obvious. Routine intramuscular immunizations can be administered safely when followed by 5 minutes of pressure to the site. Large intramuscular injections should be avoided. Serious and permanent muscular damage is not frequent among children who have hemophilia. Tendon shortening can occur, but it usually is the result of more chronic and severe hemarthroses, due to peripheral nerve damage, or caused by limited mobility for several days or longer following inadequately treated soft-tissue or joint bleeding. References: DiMichele D. Hemophilia 1996. New approach to an old disease. Pediatr Clin North Am. 1996;43:709-736 Montgomery RR, Cox Gill J, Scott JP. Hemophilia and von Willebrand disease. In: Nathan DG, Orkin SH, Oski FA, Ginsburg D, eds. Nathan and Oskis Hematology of Infancy and Childhood. 5th ed. Philadelphia, Pa: WB Saunders Co; 1998:1631-1644 Rodgers GM, Greenberg CS. Inherited coagulation disorders. In: Lee GR, Foerster J, Lukens J, Paraskevas F, Greer JP, Rodgers GM, eds. Wintrobes Clinical Hematology. 10th ed. Baltimore, Md: Williams & Wilkins; 1999:1682-1692 Critique 26. Preferred Response: D

[View Question] Use of recombinant erythropoietin (rhEPO) to improve chronic anemia has increased in several different disorders. Erythropoietin is a glycoprotein synthesized predominantly in the kidney. Its production is induced by hypoxia, and its primary effect is to stimulate erythroid progenitor cell proliferation. It was the first hematopoietic growth factor to be identified experimentally. Initially rhEPO was used in patients who had chronic renal failure and could not produce adequate amounts of this cytokine. Administration of rhEPO parenterally three times a week allows most patients on dialysis to become transfusion-independent. Dosages have varied, but many centers start dialysis-dependent patients at 50 to 100 U/kg of body weight subcutaneously or intravenously three times a week. Subcutaneous administration

may be more efficacious. The role of rhEPO in patients who have renal failure that does not yet require dialysis is less well defined. The primary side effects of rhEPO in chronic renal failure are the development of iron deficiency (related to increased iron utilization), hypertension, and seizures. The administration of iron helps to prevent iron deficiency. The hypertension probably is due to reversal of the vasodilatory effects of the anemia and occurs in only a minority of patients. The etiology of the seizures is not clear and may not differ from that seen in patients who have chronic renal failure and are not receiving rhEPO. rhEPO also has been used in the treatment of anemia associated with chronic disease. Although concentrations of erythropoietin in affected patients may be normal, they are not elevated appropriately for the degree of anemia. rhEPO can improve anemia in patients who have chronic inflammatory disorders such as rheumatoid arthritis. Its efficacy also has been demonstrated in children infected with human immunodeficiency virus. These patients frequently develop anemia because of the chronic illness, opportunistic infections, and erythroid suppression from antiviral therapy, particularly zidovudine. rhEPO may increase hemoglobin concentrations and improve the patients overall well-being when endogenous erythropoietin concentrations are below 500 U/L. rhEPO also may play a role in the prevention of anemia among children receiving chemotherapy. Both the hemoglobin concentration and the patients overall well-being can be improved, particularly in those receiving platinum analogues. The initial studies investigating rhEPO use were performed in adults, but data on its use in children are accumulating. One issue is cost-effectiveness; transfusions are considerably less expensive than rhEPO, but their safety remains a concern. rhEPO has been evaluated in the treatment of anemia of prematurity. Erythropoietin concentrations in affected infants are not appropriately elevated for the severity of anemia. rhEPO results in a more rapid reticulocyte response, but its overall benefit in this otherwise self-limited disorder remains controversial. Patients who have bone marrow failure, particularly myelodysplastic syndromes, also have benefited from rhEPO, but these disorders are uncommon in children, and pediatric experience is limited. rhEPO also plays a preoperative role in certain situations. Erythropoietin can increase the size of autologous preoperative red cell donations, and it is useful pre- and postoperatively in patients who refuse to receive blood, such as Jehovahs Witnesses.

rhEPO is important therapeutically because it is specific in stimulating erythropoiesis. Iron, folate, and copper are effective in managing specific deficiencies of these substances, but they do not enhance red blood cell production. Red blood cell transfusions are useful in patients such as the girl in the vignette, but rhEPO can limit their use. References:

Abshire TC. The anemia of inflammation. A common cause of childhood anemia. Pediatr Clin North Am. 1996;43:623-637 Brugnara C, Platt OS. The neonatal erythrocyte and its disorders. In: Nathan DG, Orkin SH, Oski FA, Ginsburg D, eds. Nathan and Oskis Hematology of Infancy and Childhood. 5th ed. Philadelphia, Pa: WB Saunders Co; 1998:19-52 Gordon MS, Sosman JA. Clinical application of cytokines and biologic response modifiers. In: Hoffman R, Benz EJ Jr, Shattil SJ, Furie B, Cohen HJ, Silberstein LE, McGlave P, eds. Hematology: Basic Principles and Practice. 3rd ed. New York, NY: Churchill Livingstone; 2000:946-947 Critique 47. Preferred Response: E

[View Question] The triad of recurrent infections, bleeding (due to thrombocytopenia), and eczema (Figure 47A) reported for the boy in the vignette is characteristic of Wiskott-Aldrich syndrome (WAS), an X-linked recessive disorder. Affected children generally come to medical attention because of repeated episodes of pneumonia or otitis media due to Streptococcus pneumoniae or Haemophilus influenzae. However, sepsis or meningitis due to these organisms or infections caused by opportunistic organisms also may occur. Predisposition to infection is the result of a number of defects in both humoral and cell-mediated immunity. Patients who have WAS often have platelet counts in the range of 15 to 30 x 103/cu mm (15 to 30 x 109/L), and their platelets are abnormally small. As a result, they experience bleeding that, at times, may be life-threatening (eg, intracranial hemorrhage). The cutaneous findings are typical of atopic dermatitis, although petechiae may be observed. Acrodermatitis enteropathica is an autosomal recessive disorder of zinc transport that presents in infancy with diarrhea, failure to thrive, and rash. In contrast to WAS, the eruption (Figure 47B) of acrodermatitis enteropathica consists of erythematous, crusted patches around the mouth and eyes, on the distal extremities, and in the perineum (Figure 47C). Patients who have ataxia telangiectasia (an autosomal recessive disorder) may be distinguished from those who have WAS by the presence of severe cerebellar ataxia and telangiectasias (Figure 47D) involving the conjunctivae (Figure 47E) and skin. Other cutaneous manifestations include vitiligo, caf au lait spots, and premature graying of the hair. Although the lesions of atopic dermatitis are analogous to those seen in WAS, patients who have the latter disorder often exhibit petechiae and the associated abnormalities discussed previously. Langerhans cell histiocytosis (LCH) is the term applied to a group of disorders characterized by the accumulation of histiocytes in the skin or other organs. Unlike the lesions of WAS, those of LCH are erythematous, orange or yellow-brown papules or nodules. Individual lesions may exhibit scale or petechiae or may become purpuric (Figure 47F). Affected individuals may present with skin disease alone or systemic complications (eg, bone pain or swelling, diabetes insipidus).

References: Hong R. Combined immunodeficiency diseases. In: McMillan JA, DeAngelis CD, Feigin RD, Warshaw JB, eds. Oskis Pediatrics: Principles and Practice. 3rd ed. Philadelphia, Pa: Lippincott Williams & Wilkins; 1999:2092-2099 Weston WL, Lane AT, Morelli JG. Color Textbook of Pediatric Dermatology. 2nd ed. St Louis, Mo: Mosby, Inc; 1996:200-202, 313-314 Critique 90. Preferred Response: D

[View Question] The most common site of relapse of acute lymphoblastic leukemia (ALL) is the bone marrow, but relapses in the central nervous system and the testes also are important. Because more than 95% of treated children attain an initial remission, most treatment failures are due to later relapses. Marrow relapse usually is identified by unexpected decreases in hemoglobin and platelet counts. Absolute neutrophil counts typically fall, but the total white blood cell count may rise if blasts (Figure 90A) appear in the peripheral blood. If there is a suspicion of potential relapse, the bone marrow should be examined. Central nervous system relapses most often present with signs and symptoms of increased intracranial pressure, including headache, nausea and vomiting, irritability, nuchal rigidity, and papilledema, as well as occasional focal cranial nerve involvement. The appearance of such signs warrants lumbar puncture. Although intracranial pressure usually is increased, a lumbar puncture is relatively safe because the pressure is distributed evenly throughout the cerebrospinal fluid. All boys who have ALL must undergo testicular examination during routine health supervision visits. Testicular relapse presents with painless testicular enlargement that most often is unilateral and frequently is missed by the child and parents. Both central nervous system and testicular relapse occur when the bone marrow remains in remission. Relapse can occur during chemotherapy or after its cessation. Children who relapse while receiving therapy or within 6 months of the cessation of chemotherapy are unlikely to sustain a prolonged second remission. Children who have classic pre-B-cell ALL and relapse more than 6 months after cessation of therapy may experience prolonged remissions, and an increasing number may be cured. The risk of relapse is highly dependent on the prognostic features identified at the time of diagnosis. Poor prognostic factors include higher white blood cell counts, age (<2 y or >10 y), the presence of specific cytogenetic abnormalities (such as t(8,14) and t(9,22) translocations), and immunophenotyping (mature T- or B-cell phenotypes having a worse prognosis than the pre-B-cell phenotype). Initially, children who had poor prognostic features had much higher relapse rates than children who had standard risk features. Treatment protocols have focused on intensifying therapy for high-risk children, and survival differences between high-risk and

standard-risk children have narrowed. Approximately 85% of affected children now are surviving their disease. A specific example of improved survival is the elimination of gender as a risk factor. Originally, males fared worse than females, but by prolonging the therapy of boys to 3 years instead of the 2 years of treatment used for girls, survival rates are now equivalent. Bony pain, lymphadenopathy, hepatosplenomegaly, and tonsillar enlargement are less common as presenting signs of relapse than they are at the time of initial diagnosis. References: Berg SL, Steuber CP, Poplack DG. Clinical manifestations of acute lymphoblastic leukemia. In: Hoffman R, Benz EJ Jr, Shattil SJ, et al, eds. Hematology: Basic Principles and Practice. 3rd ed. New York, NY: Churchill Livingstone; 2000:1070-1078 Niemeyer CM, Sallan SE. Acute lymphoblastic leukemia. In: Nathan DG, Orkin SH, Oski FA, Ginsburg D, eds. Nathan and Oskis Hematology of Infancy and Childhood. 5th ed. Philadelphia, Pa: WB Saunders Co; 1998:1245-1285 Pui CH. Acute lymphoblastic leukemia. Pediatr Clin North Am. 1997;44:831-846 Critique 112. Preferred Response: B

[View Question] Most children who develop deep vein thrombosis or pulmonary emboli have hereditary thrombophilia, defined as a genetically determined increased risk of thrombosis. Affected patients may have recurrent thromboses or family histories of single or multiple episodes of deep vein thromboses or pulmonary emboli and less commonly, thromboses involving other sites. The inherited thrombotic disorders are associated almost exclusively with venous thrombosis. The most prevalent of these disorders have been recognized in the past 10 years: factor V Leiden (a defect of factor V that resists the antithrombotic effects of activated protein C) and prothrombin G20210A (a genetic defect in which the altered prothrombin molecule is more coagulable). The next most prevalent condition is hyperhomocystinemia, which has both familial and acquired components. Deficiencies of protein C, protein S, and antithrombin III constitute the other common hypercoagulable conditions. Acquired events may precipitate thromboses, including immobilization after surgery or trauma, pregnancy, malignancy, estrogen administration (including oral contraceptive pills), nephrotic syndrome, diabetes mellitus, hyperviscosity, infusion of prothrombin complex concentrates, and thrombotic thrombocytopenic purpura. The increasing use of indwelling venous catheters is associated with a high risk of thromboses, especially in the femoral vein. The presence of an acquired circulating lupus anticoagulant is an important precipitating condition, but this is less common in children than adults. The presence of one of these acquired risk factors for thrombosis does not exclude the possibility of hereditary thrombophilia. Some of the familial disorders are quite common; 6% of Caucasian Americans are estimated to have factor V Leiden and 2% of the population of the Netherlands has

prothrombin G20210A. Many, if not most, of these individuals only develop thromboses in the presence of a secondary procoagulant trigger, such as pregnancy, surgery, immobilization, estrogen use, or even a second form of hereditary thrombophilia. In one study, 16% of patients had more than one type of thrombophilia. The most common sites of thromboses in hereditary thrombophilia are the deep leg veins, iliac veins, and pulmonary arteries. Thromboses in mesenteric, portal, and cerebral veins have been described but are less common. The initial laboratory evaluation of children who have thromboses should include a complete blood count, review of the peripheral smear, evaluation of hepatic and renal function, and urinalysis. Specific assays for thrombophilia should include prothrombin time (PT) and partial thromboplastin time (PTT), a functional assay of activated protein C resistance or analysis of the factor V Leiden mutation, prothrombin G20210A analysis, plasma homocysteine concentration, protein C activity (functional assay), protein S activity assay (or free protein S antigen), antithrombin III activity, fibrinogen measured by a clottable assay, and thrombin time. Additional tests for antiphospholipid antibodies should be performed to exclude the possibility of this acquired disorder. Elevations of factor VIII and of Lp(a) may increase the risk, and these factors can be measured. It is best to evaluate patients when they are not receiving oral anticoagulants, but if this is not possible, only proteins C and S will be altered dramatically. If necessary, these can be measured by substituting a heparin anticoagulant for 2 weeks prior to drawing blood, by studying family members, or by comparing the levels of other vitamin K-dependent factors. Acute thromboses can decrease the levels of antithrombin III and proteins C and S transiently. Management of thromboses in patients who have hereditary thrombophilia is complex and complicated by the rarity of these events in pediatrics. Assessment of longterm risk, including a history of two or more spontaneous thromboses, a single lifethreatening thrombosis, a single thrombosis at an usual site (such as cerebral or mesenteric thrombosis), and a single thrombosis in the presence of more than one biologic defect, may require indefinite anticoagulation. These are important decisions because the prolonged use of oral anticoagulants can be more difficult to regulate in children, and the risks of their use are higher. Children who are asymptomatic or who have had a single thrombosis associated with a transient prothrombotic stimulus do not require prolonged anticoagulation but may merit prophylaxis during some high-risk situations. Compression of the femoral vein due to indwelling catheters is associated with a high risk of thromboses, but the use of catheters in children is uncommon, and the compression is recognized easily. Androgenic steroid use should not precipitate thromboses. References: Bauer KA. Hypercoagulable states. In: Hoffman R, Benz EJ Jr, Shattil SJ, et al, eds. Hematology: Basic Principles and Practice. 3rd ed. New York, NY: Churchill Livingstone; 2000:2009-2039

Goodnight SH, Griffin JH. Hereditary thrombophilia. In: Beutler E, Lichtman MA, Coller BS, Kipps TJ, Seligsohn U, eds. Williams Hematology. 6th ed. New York, NY: McGraw-Hill; 2001:1969-1707 Critique 134. Preferred Response: E

[View Question] Vascular disorders may be responsible for petechiae or purpura (Figure 134A) in children who have normal or increased platelet counts. Meningococcemia and other bacterial infections are critical causes of vascular purpura because of the high associated morbidity and mortality and the critical need to consider the diagnosis and begin therapy rapidly. The purpura is due to either direct vascular invasion by the organism or to an endotoxin-induced Schwartzman reaction. Clinically, erythematous papules are seen initially, but they soon evolve into widespread petechiae with purple to slate-gray purpura. Disseminated intravascular coagulation (DIC) develops less frequently, causing petechiae and purpura due to both thrombocytopenia and acrocyanosis that progresses to symmetric peripheral gangrene following fibrin deposition. Patients who have DIC are very ill-appearing and in septic shock. Laboratory studies reveal thrombocytopenia, hypofibrinogenemia, and prolongation of the partial thromboplastin time and the prothrombin time. Other infectious etiologies of vascular purpura include gram-negative sepsis (particularly in association with granulocytopenia or immune suppression), scarlet fever, Haemophilus influenzae infection, Streptococcus pneumoniae infection, Rocky Mountain spotted fever (Figure 134B), and disseminated fungal infections. Viral infections are the most common causes of vascular purpura. Although the purpura occasionally can be due to specific viruses, such as parvovirus B19 or Hanta virus causing hemorrhagic fever, most often it occurs in otherwise well children who have a variety of more routine viral illnesses. The critical clinical issue is to exclude the more serious bacterial infections from the differential diagnosis before assuming a viral etiology. Purpura can be seen in a variety of other illnesses, including glucocorticoid excess, Ehlers-Danlos syndrome, and serum sickness. Henoch-Schnlein purpura (Figure 134C) is the most common cause of noninfectious vascular purpura in children. The palpable purpuric lesions are distributed classically on the lower extremities, avoiding the trunk and head. Collagen-vascular disorders also may present with a purpuric rash. The classic blueberry muffin (Figure 134D) baby has a vasculitis that most often is due to congenital toxoplasmosis, rubella, or cytomegalovirus. Hereditary hemorrhagic telangiectasia (Osler-Weber-Rendu syndrome) is an autosomal dominant disorder that usually presents with recurrent epistaxis that worsens with age. The most severely affected patients develop epistaxis during childhood, with cutaneous changes beginning to appear at puberty.

The lesions of vascular purpura are usually palpable. In contrast, the petechiae and purpura of platelet dysfunction typically are nonpalpable. Platelet dysfunctions must be considered in patients who have purpura and normal platelet counts. Clinically significant platelet dysfunctions in children usually are due to a variety of congenital disorders. The clinical history and physical examination generally will allow differentiation of platelet dysfunction and vascular purpura, with the latter usually associated with a more specific rash. Platelet function studies occasionally may be necessary to document a platelet disorder. References: Baselga E, Drolet BA, Esterly NB. Purpura in infants and children. J Am Acad Dermatol. 1997;37:673-705 Schneiderman P. The vascular purpuras. In: Beutler E, Lichtman MA, Coller BS, Kipps TJ, eds. Williams Hematology. 5th ed. New York, NY: McGraw-Hill; 1995:1401-1412 Critique 156. Preferred Response: E

[View Question] Transient erythroblastopenia of childhood (TEC) is the most common form of pure red blood cell aplasia in children. This acquired disorder occurs in previously healthy children who present with severe anemia. Its pathophysiology is not understood, although many have presumed a viral etiology. Most patients are between 1 and 3 years of age. The average hemoglobin at presentation is 5.7 g/dL (57 g/L). White blood cell and platelet counts are usually normal, but neutropenia is common. The anemia is normocytic and normochromic, and hemoglobin F levels are normal for age. Severe erythroid hypoplasia is evident in the bone marrow unless spontaneous recovery already is occurring, in which case erythroid hyperplasia precedes the reticulocytosis and recovery. All patients recover, with 60% requiring a transfusion for support until recovery. The recommendation is to observe patients and provide transfusion if there is a risk of cardiovascular compromise. No other therapy is indicated. TEC must be distinguished from Diamond-Blackfan anemia (DBA), a much more serious disorder. Both dominant and autosomal recessive patterns of inheritance have been reported for DBA, but 75% of cases are sporadic, suggesting that they are acquired due to new mutations or to variable penetrance. A defect of the erythroid stem cell is most likely responsible for the disorder. Pallor usually is recognized at or soon after birth. Approximately 25% of patients demonstrate physical anomalies, most commonly of the head and face. Hemoglobin levels average 4 g/dL (40 g/L) at diagnosis. Macrocytosis is frequent, and reticulocytes are decreased or absent. White blood cell counts are usually normal. Although platelet counts also generally are normal, thrombocytopenia may be noted at some point in 25% of patients. Elevated hemoglobin F levels, increased titers of red blood cell membrane i antigen, and higher mean corpuscular volumes are consistent with more fetal-like erythropoiesis. Bone marrow examination usually reveals erythroid hypoplasia. Red blood cell adenosine deaminase levels often are elevated.

Unlike TEC, DBA does not remit spontaneously. Corticosteroids are useful. Fewer than 5% of patients respond rapidly and enter a steroid-independent remission, up to 60% respond but remain steroid-dependent, up to 20% eventually may be able to discontinue steroids, fewer than 5% respond only to very large doses of steroids and become transfusion-dependent, and 30% to 40% fail to respond to corticosteroids. Approximately 15% of patients die. Bone marrow transplantation can be effective. There also appears to be a small but increased risk of leukemia among children who have DBA. One feature that helps to distinguish TEC from DBA is age, with only 20% of TEC patients being younger than 1 year of age and more than 90% of children who have DBA being diagnosed by 1 year. The presence of associated physical anomalies, elevated hemoglobin F, macrocytosis, increased red blood cell i antigen, and elevated red blood cell adenosine deaminase support a diagnosis of DBA. Other forms of pure red blood cell aplasia in children are very rare. The anemia of chronic disease usually is associated with an obvious underlying disorder and rarely is profound. Fanconi anemia seldom presents with anemia in the first year, and usually physical anomalies, pancytopenia, and macrocytosis are evident. Severe iron deficiency would be expected to be microcytic. References: Alter BP, Young NS. The bone marrow failure syndromes. In: Nathan DG, Orkin SH, Oski FA, Ginsburg D, eds. Nathan and Oskis Hematology of Infancy and Childhood. 5th ed. Philadelphia, Pa: WB Saunders Co; 1998: 286-301 Freedman MH. Inherited forms of bone marrow failure. In: Hoffman R, Benz EJ Jr, Shattil SJ, et al, eds. Hematology: Basic Principles and Practice. 3rd ed. New York, NY: Churchill Livingstone; 2000:260-297 Critique 178. Preferred Response: D

[View Question] Approximately 80% of patients who have Fanconi anemia exhibit congenital anomalies. Recognizing these anomalies is important because the aplastic anemia usually is not evident until 4 to 14 years of age. Pancytopenia at birth is unusual. Among the anomalies are hyperpigmentation (60% of patients); short stature (57%); upper limb anomalies (48%) that include absent, hypoplastic, supernumerary, or bifid thumbs and aplasia of the first metacarpal or the radius; hypogonadism (40%); and microcephaly (27%). Almost 20% of affected patients manifest a variety of other skeletal anomalies. The only potential cure for this autosomal recessive disorder is bone marrow transplantation, and early diagnosis allows for better donor identification and patient preparation. It is also important to make the diagnosis to enable parents to use prenatal diagnosis for future children. Several complementation groups have been identified with Fanconi anemia, with type A accounting for almost 66% of cases. If the specific complementation groups can be identified in the index case, DNA analysis can be used for prenatal diagnosis. If a specific DNA defect is not identified, increased chromosomal breakage in peripheral blood lymphocytes in the presence of cross-linking agents can be used.

Several other disorders are associated with skeletal anomalies and hematologic disease. Shwachman-Diamond syndrome is an autosomal recessive disorder of exocrine pancreatic function and variable hematologic abnormalities, including neutropenia, red blood cell hypoplasia, thrombocytopenia, pancytopenia, and myelolymphoproliferative disease. Skeletal abnormalities include metaphyseal dysplasia, long bone trabeculation, short or flared ribs, thoracic dystrophy, and clinodactyly (Figure 178A). Some patients present at birth with thoracic dystrophy; others have short, flared ribs. The metaphyseal dysostosis of the long bones may not be detectable until after 12 months of age. Shwachman-Diamond syndrome and Fanconi anemia share the characteristics of bone marrow dysfunction and growth failure, but the pancreatic insufficiency and malabsorption are unique to the former, and the skeletal anomalies differ. Dyskeratosis congenita is a rare inherited bone marrow failure syndrome in which aplastic anemia complicates 50% of cases and in which there is a predisposition to malignancies. The inheritance is complex. Skin and nail changes often appear during childhood. Skeletal abnormalities, which include osteoporosis, avascular necrosis, abnormal bone trabeculation, scoliosis, and mandibular hypoplasia, are seen in 20% of cases. The skin and nail abnormalities are distinct from the changes seen in Fanconi anemia. Thrombocytopenia and absent radii (TAR) syndrome is also known as congenital amegakaryocytic thrombocytopenia. The classic finding of absent radii with thumbs present differentiates it from Fanconi anemia. Anomalies of hands, shoulders, and lower limbs also may be seen. TAR presents with isolated thrombocytopenia in the neonate, but 45% of patients ultimately develop aplastic anemia. Inheritance is complex but usually autosomal recessive. Diamond-Blackfan anemia, or congenital hypoplastic anemia, is associated with physical abnormalities in approximately 25% of patients. Anomalies include a typical facies, micrognathia, flattening of the thenar eminences, and triphalangeal thumbs. Bloom syndrome is a rare chromosomal breakage syndrome in which several skeletal anomalies may be found; the major hematologic risk is development of leukemia and other malignancies. Only mild anemia is typical. Other rare disorders associated with both skeletal anomalies and hematologic disorders include familial aplastic anemia syndromes, cartilage-hair hypoplasia, Dubowitz syndrome, Seckel syndrome, and Poland syndrome (Figure 178B). Acquired disorders also may be associated with both hematologic and skeletal abnormalities. Children who have newly diagnosed acute lymphoblastic leukemia often present with bone pain and generalized radiologic signs, but they do not have congenital abnormalities. References: Alter BP, Young NS. The bone marrow failure syndromes. In: Nathan DG, Orkin SH, Oski FA, Ginsburg D, eds. Nathan and Oskis Hematology of Infancy and Childhood. 5th ed. Philadelphia, Pa: WB Saunders Co; 1998:238-335

Freedman MH. Inherited forms of bone marrow failure. In: Hoffman R, Benz EJ Jr, Shattil SJ, et al, eds. Hematology: Basic Principles and Practice. 3rd ed. New York, NY: Churchill Livingstone; 2000:260-297 Critique 201. Preferred Response: C

[View Question] Neuroblastoma usually presents as a nontender, firm, fixed abdominal mass in children younger than 5 years of age. The family often identifies the mass, and such a complaint never should be ignored. With the occasional exception of a child who has constipation, this finding is almost always a true mass. If the child is uncooperative and the clinician cannot readily perform a thorough abdominal examination, this complaint alone is an indication for abdominal ultrasonography. The primary site of neuroblastoma most commonly is the adrenal gland. Ultrasonography reveals a suprarenal mass displacing the kidney downward but not altering its intrinsic architecture, as described for the child in the vignette. Calcifications are present in approximately 50% of cases. Other primary sites, in order of decreasing frequency, are the paravertebral retroperitoneum, posterior mediastinum, pelvis, and cervical area. The presenting abdominal mass also may represent hepatic metastases, which can be massive. Metastatic disease may be noted in several other sites, including skin, bone marrow, bone, and periorbital tissue. Horner syndrome can be a complication of high thoracic and cervical masses. These children often appear chronically ill. Approximately 90% to 95% of neuroblastomas produce increased catecholamines, which can be measured by their urinary metabolites. Urinary vanillylmandelic acid (VMA) and homovanillic acid (HVA) are measured and compared to the excretion of creatinine in the sample. This allows the use of a spot sample and avoids the need for timed 24-hour collections that are very difficult to obtain. Adrenocortical carcinomas involve the adrenal gland, but they are much less common than neuroblastomas. Wilms tumor is another common malignancy that almost always presents with abdominal swelling or the presence of an abdominal mass. Affected children frequently are otherwise well. Ultrasonography will identify a solid intrarenal mass. Involvement of the renal veins and inferior vena cava, as well as the contralateral kidney, should be noted. Hepatoblastoma is less common and usually presents with abdominal distention and hepatomegaly; a discrete tumor mass generally is not palpable. The mass may move more with inspiration than do renal or adrenal masses. It may be difficult to differentiate hepatoblastoma initially from neuroblastoma, lymphoma, or leukemia with hepatic involvement. Serum levels of alpha-fetoprotein are elevated in 80% to 90% of affected children. Other malignant abdominal masses include hepatocellular carcinoma, teratocarcinoma, and adrenal carcinoma. Rhabdomyosarcoma usually presents in the pelvis, involving the bladder, uterus, vagina, or testes; less frequently it appears as a diffuse retroperitoneal mass. It does not involve the adrenal gland.

Most abdominal masses are not malignant. The most common renal mass is congenital hydronephrosis, and multicystic-dysplastic kidney also is frequent. Choledochal cysts, bezoars, gastrointestinal duplications, and polyps present as masses. Ultrasonography readily differentiates most of these benign masses from malignant ones and is the initial imaging study of choice in children who have abdominal masses. References: Anveden-Hertzberg L, Gauderer MW. Abdominal masses. In: Kliegman RM, Nieder ML, Super DM, eds. Practical Strategies in Pediatric Diagnosis and Therapy. Philadelphia, Pa: WB Saunders Co; 1996:387-399 Brodeur GM, Castleberry RP. Neuroblastoma. In: Pizzo PA, Poplack DG, eds. Principles and Practice of Pediatric Oncology. 3rd ed. Philadelphia, Pa: Lippincott-Raven; 1997:761-797 Castleberry RP. Biology and treatment of neuroblastoma. Pediatr Clin North Am. 1997;44:919-937 Critique 223. Preferred Response: A

[View Question] Acute lymphoblastic leukemia (ALL) and aplastic anemia usually present with purpura and pallor, and it is critical to be able to differentiate between these life-threatening disorders. ALL is much more common and often presents with additional signs and symptoms. Hepatosplenomegaly occurs in more than 60% of patients; lymphadenopathy in more than 50%; and bone pain, onset of a limp, or refusal to walk in as many as 40%. Arthralgias arising from joint infiltration can imitate juvenile rheumatoid arthritis. Fever occurs in more than 50% of children and is more often due to the disease than to secondary infection. Respiratory distress due to superior vena cava syndrome is seen, particularly in adolescents. Occasional patients present with symptoms of increased intracranial pressure due to central nervous system involvement. An elevated white blood cell count, almost always associated with blasts in the peripheral blood, is an obvious sign of leukemia. However, as many as 50% of patients have a normal white blood cell count at diagnosis. Those who have the lowest counts may not demonstrate blasts in the peripheral blood, making the differentiation from aplastic anemia difficult, especially in the absence of other clinical signs. Children who have ALL and low white blood cell counts have lower tumor burdens. They have less lymphadenopathy and organomegaly and are more likely to have normal uric acid levels. In children who report bone pain, radiographs may identify typical leukemic changes, especially around the knees, wrists, and ankles, but the pain may be present without radiologic findings. Bone marrow examination may be essential to differentiate leukemia and aplastic anemia. A bone marrow aspirate usually establishes the diagnosis of leukemia. The marrow is examined with routine morphology, special histochemical staining, analysis with flow cytometry to help define the lineage of leukemic cells, and chromosomal analysis. A bone marrow biopsy defines the decrease in marrow cellularity expected in aplastic anemia. In

occasional children, it is not possible to obtain a marrow aspirate, and marrow biopsy becomes essential to differentiate leukemia from aplastic anemia. Acute myelogenous leukemia (AML) accounts for 15% to 20% of childhood leukemias. The clinical presentation is similar to ALL, but skin, gum, and central nervous system involvement are more common. Patients who have high white blood cell counts can develop life-threatening central nervous system disease due either to hemorrhage or cerebral leukostasis. Although estimates vary considerably, the incidence of leukemia is probably at least 20-fold greater than that of aplastic anemia. Aplastic anemia most commonly is acquired, but the etiology is not known. The only signs and symptoms in affected children relate to the anemia, thrombocytopenia, and neutropenia secondary to bone marrow failure. A smaller number of children are shown to have exposure to drugs (eg, chloramphenicol, sulfonamides), chemicals (benzene, insecticides), infections (hepatitis, Epstein-Barr virus), and other disorders (paroxysmal nocturnal hemoglobinuria). Rarely, children may present with transient aplastic anemia, only to develop ALL within a few months. Inherited bone marrow failure syndromes are an important consideration because manifestations of these disorders may be apparent upon physical examination. Fanconi anemia, an autosomal recessive disorder, is most common. Approximately 80% of affected patients manifest physical abnormalities, including hyperpigmentation, short stature, abnormal upper limbs (particularly thumbs), hypogonadism, a small face, and typical facies. Although the physical findings are evident at birth, the pancytopenia usually is not evident until an average age of 8 years. Several other congenital disorders have been associated with bone marrow failure, but these are very rare. They include dyskeratosis congenita, Shwachman-Diamond syndrome, and reticular dysgenesis. Myelodysplastic syndromes are uncommon in children, but at presentation they can be confused easily with leukemia or aplastic anemia. Affected patients may present with involvement of any single or multiple cell lines. Clues to the underlying diagnosis include the presence of nucleated erythrocytes, tear drop-shaped erythrocytes, and relatively small numbers of immature white blood cells in peripheral blood. The bone marrow is hypercellular, with disordered and morphologically abnormal maturation of cell lines. Approximately 50% of patients have a clonal chromosomal abnormality that often involves chromosome 7. Most affected children progress to leukemia or die of bone marrow failure. Parvovirus infection causes transient erythroid hypoplasia, but this usually becomes clinically significant only in patients who have underlying hemolytic anemias. References: Alter BP, Young NS. The bone marrow failure syndromes. In: Nathan DG, Orkin SH, Oski FA, Ginsburg D, eds. Nathan and Oskis Hematology of Infancy and Childhood. 5th ed. Philadelphia, Pa: WB Saunders Co; 1998:238-335

Freedman MH. Inherited forms of bone marrow failure. In: Hoffman R, Benz EJ Jr, Shattil SJ, et al, eds. Hematology: Basic Principles and Practice. 3rd ed. New York, NY: Churchill Livingstone; 2000:260-297 Pui CH. Acute lymphoblastic leukemia. Pediatr Clin North Am. 1997;44:831-846 Critique 245. Preferred Response: A

[View Question] Providing genetic counseling to families that have severe factor VIII deficiency hemophilia (hemophilia A) has greatly improved, but it is limited by the absence of any family history in 30% of newly diagnosed children. This is due primarily to the strong predisposition of the factor VIII gene to mutate. The new mutations are usually inversions in specific locations and occur almost exclusively during male meiosis. Because the mutation rarely occurs in female germ cells, nearly all mothers of patients who have inversions are carriers of the disease. A less frequent explanation for the absence of a positive family history is the random absence of affected males in recent generations. Inversions are the most common single gene defects responsible for the clinically severe forms of factor VIII deficiency hemophilia. Approximately 50% to 60% of patients who have hemophilia have the severe form of the disease, with factor VIII levels of less than 1%. Patients who have moderate and mild hemophilia (factor VIII levels of 1% to 5% and greater than 5%, respectively) have different genetic defects. More than 200 specific genetic defects now have been associated with hemophilia. The gene for factor VIII is located near the terminus of the long arm of the X chromosome. Therefore, males are affected almost exclusively. Hemophiliac males do not transmit the disease to their sons, but their daughters are obligate and usually asymptomatic carriers. Carriers usually can be detected easily. Prenatal diagnosis is not possible in the 30% of patients who have no family history. However, once the disease has been recognized, prenatal diagnosis can be provided to almost all families. DNA analysis for the common inversion is now available and provides a precise genetic diagnosis in 45% of patients who have severe disease. The remaining 55% of patients have a variety of defects, but DNA sequence screening may not be available. However, accurate genetic diagnosis that allows carrier testing and prenatal diagnosis is achieved in more than 90% of families by distinguishing the polymorphic DNA sequence difference between the two copies of the potential carriers X genes. If the two X genes can be distinguished, analysis of the affected male will determine which carries the mutation without identifying the precise mutation. Prenatal diagnosis then can be performed as early as 10 weeks of pregnancy using chorionic villus biopsy sampling. Prior to the availability of DNA testing, the only option was direct fetal cord blood sampling and testing for factor VIII clotting activity. Although accurate, this approach can be performed only late in pregnancy and is associated with a risk of fetal loss. However, this approach still may be necessary in families in which no informative DNA markers are available.

Factor VIII deficiency hemophilia is approximately five times more frequent than factor IX deficiency hemophilia (hemophilia B), affecting approximately 1 in 5,000 male births. Both disorders have no apparent racial or ethnic predilection. The factor IX gene is considerably smaller (34 kb in length) and less complex than the factor VIII gene. More than 400 genetic defects causing this disease have been identified. Virtually all factor IX-deficient patients have an identifiable DNA defect that can be applied readily to carrier detection and prenatal diagnosis. Because there is a very low spontaneous mutation rate in this form of hemophilia, the family history almost always is positive. References: Bell B, Canty D, Audet A. Hemophilia: an updated review. Pediatr Rev. 1995;16:290-298 DiMichele D. Hemophilia 1996. New approach to an old disease. Pediatr Clin North Am. 1996;43:709-736 Montgomery RR, Cox Gill J, Scott JP. Hemophilia and von Willebrand disease. In: Nathan DG, Orkin SH, Oski RA, Ginsburg D, eds. Nathan and Oskis Hematology of Infancy and Childhood. 5th ed. Philadelphia, Pa: WB Saunders Co; 1998:1632-1644 Rodgers GM, Greenberg CS. Inherited coagulation disorders. In: Lee GR, Foerster J, Lukens J, Parasekevas F, Greer JP, Rodgers GM, eds. Wintrobes Clinical Hematology. 10th ed. Baltimore, Md: Williams & Wilkins; 1999:1683-1701 Critique 264. Preferred Response: B

[View Question] The partial thromboplastin time (PTT) and prothrombin time (PT) are important screening tests in the evaluation of children who have bleeding disorders. The PTT measures all the factors in the pathway except factor VII. The PTT not only measures more factors than the PT, but it measures those most commonly involved in coagulopathies, such as factor VIII, which is involved in von Willebrand disease and factor VIII deficiency hemophilia (hemophilia A). von Willebrand disease is overwhelmingly the most common coagulopathy, affecting as many as 1% of the normal population. Factor VIII deficiency hemophilia is clinically much more severe, but it only affects 1 in 5,000 births. The PTT also measures factor IX and helps to identify patients who have factor IX deficiency hemophilia (hemophilia B). Circulating antiphospholipid antibodies (lupus anticoagulant) are more likely to prolong the PTT. Finally, the PTT is much more sensitive to prolongation from heparin. Normal ranges for the PTT vary among laboratories but usually are in the range of 20 to 35 seconds for both children and adults. The PTT is prolonged in newborns, particularly in preterm infants. The PTT usually remains normal until the level of an affected factor falls below 30%, with a normal value arbitrarily set at 100% and the normal range approximately 50% to 150%. The PT measures factor VII as well as the factors common to both the PT and PTT (factors X, V, II, and fibrinogen). The PT usually is in the range of 10 to 13 seconds and

typically becomes prolonged when the level of involved factor is below 40%. Prolongation of the PT in the presence of a normal PTT usually reflects isolated factor VII deficiency. The PT is more useful in monitoring the effect of coumarin-type anticoagulants. Although these drugs prolong both tests, the PT reflects changes more rapidly. The half-life of factor VII, at 4 to 6 hours, is much shorter than the other coagulation factors. Therefore, changes in factor VII levels induced by the coumarins are reflected most quickly by the PT. The International Normalized Ratio (INR) is derived from the PT compared with control values and using an international sensitivity index that corrects for variation in the sensitivity of the varying reagents used to perform the PT. The INR provides a value that reduces interlaboratory variation due to both techniques and reagents used. For this reason, coumarin-induced anticoagulation is assessed using the INR. Deficiency of factor XIII is not associated with prolongation of either the PTT or the PT. Children who have this deficiency present with delayed separation of the umbilical stump, delayed bruising or hematoma formation following trauma, and poor wound healing. Specific screening tests are available when this diagnosis is suspected. References: Montgomery RR, Cox Gill J, Scott JP. Hemophilia and von Willebrand disease. In: Nathan DG, Orkin SH, Oski RA, Ginsburg D, eds. Nathan and Oskis Hematology of Infancy and Childhood. 5th ed. Philadelphia, Pa: WB Saunders Co; 1998:1631-1644 Rodgers GM, Greenberg CS. Inherited coagulation disorders. In: Lee GR, Foerster J, Lukens J, Parasekevas F, Greer JP, Rodgers GM, eds. Wintrobes Clinical Hematology. 10th ed. Baltimore, Md: Williams & Wilkins; 1999:1682-1692 2001 Self-Assessment Exercise XV. Disorders of the blood and neoplastic disorders [Return to Category List] Questions [Print Directions] Question 14. Answer.

A previously well 17-year-old girl complains of weakness, weight loss, canker sores on her tongue, and loss of sensation and positional sense in her feet over the past few months. Her family reports that she has become pale. Laboratory findings include: hemoglobin, 73 g/L (7.3 g/dL); mean corpuscular volume, 108 fL; white blood cell count, 3.1 x 109/L (3,100/mm3); platelets, 116 x 109/L (110,000/mm3); and reticulocyte count, 1%. Examination of a peripheral blood smear reveals marked anisocytosis, many large ovalocytes, and an average of six lobes per neutrophil. Of the following, the MOST likely cause of these abnormalities is A. acquired aplastic anemia

B. C. D. E.

Fanconi anemia folic acid deficiency methyltetrahydrofolate reductase deficiency pernicious anemia Answer.

Question 57.

Two days ago a 5-year-old girl undergoing chemotherapy for acute lymphoblastic leukemia spent the morning with a playmate who now has developed varicella. She has not received varicella vaccine and has no history of having varicella. Of the following, the MOST appropriate action is to A. B. C. D. E. administer varicella vaccine administer varicella-zoster immune globulin begin oral acyclovir discontinue chemotherapy for 3 weeks obtain varicella titers to determine the need for therapy

Question 106. Answer. The parents of a 5-year-old boy who has severe factor VIII deficiency hemophilia bring him in for evaluation after he fell off his bicycle. He was not wearing a helmet. He had no loss of consciousness and appears uninjured except for a small hematoma over the right side of his forehead. Of the following, the MOST appropriate next step is to A. B. C. D. E. admit for observation without therapy discharge home infuse with factor VIII obtain computed tomography treat with desmopressin

Question 152. Answer. A newborn is recognized clinically to have Down syndrome. The parents are very concerned about the disorder and its manifestations. Of the following, the MOST important hematologic/oncologic complication is A. aplastic anemia

B. C. D. E.

leukemia macrocytic anemia platelet dysfunction thrombocytosis

Question 200. Answer. A 6-year-old boy currently receiving chemotherapy for acute lymphoblastic leukemia presents with a fever of 38.8C (102F) and symptoms of an upper respiratory tract infection. He does not have an indwelling intravenous access catheter. Laboratory findings include: hemoglobin, 115 g/L (11.5 g/dL); white blood cell count, 6.4 x 109/L (6,400/mm3); and platelets, 380 x k109/L (380,000/mm3). Of the following, the MOST important study to obtain next is A. B. C. D. E. absolute neutrophil count cerebrospinal fluid analysis chest radiograph liver function studies urine culture

Question 243. Answer. A 4-week-old exclusively breastfed boy undergoing preoperative screening prior to elective surgery is found to have macrocytic anemia. The mother is a strict vegan who emigrated from India 1 year ago. Laboratory findings include: hemoglobin, 103 g/L (10.3 g/dL); mean corpuscular volume, 135 fL; white blood cell count, 4.1 x 109/L (4,100/mm3); platelets, 110 x 109/L (110,000/mm3); and reticulocyte count, 10 x 10-3 (1%). The peripheral blood smear reveals marked anisocytosis, many large ovalocytes, and hypersegmentation of the neutrophils. Of the following, the MOST likely diagnosis is A. B. C. D. E. folic acid deficiency methylmalonic aciduria pernicious anemia physiologic anemia vitamin B12 deficiency

Answers

Critique 14.

Preferred Response: E

[View Question] Pernicious anemia is an autoimmune disorder that causes destruction of the intrinsic factorproducing gastric parietal cells. The resulting deficiency of intrinsic factor prevents normal absorption of vitamin B12. In children, pernicious anemia is associated with polyendocrinopathy syndrome that affects the adrenal, parathyroid, and thyroid glands as well as the ovaries. The clinical picture can include megaloblastic anemia, gastrointestinal symptoms, and neurologic disease. Patients become fatigued because of the anemia. Gastrointestinal symptoms may predominate, including anorexia, weight loss, nausea, constipation, and glossitis with canker sores on the tongue. The neurologic symptoms, which are due to subacute combined degeneration of the spinal cord, usually present initially with decreased vibratory and positional sense that is more severe in the lower extremities, as described for the girl in the vignette. Laboratory findings include macrocytic anemia that usually is associated with leukopenia, thrombocytopenia, and reticulocytopenia. The peripheral blood smear reveals marked variation in red blood cell size (anisocytosis), large ovalocytes, and hypersegmentation of the neutrophils. Neutrophils usually have more than the usual number of three to four lobes per cell. Vitamin B12 deficiency also results from disorders that impair gastric and small intestinal absorption and, rarely, from inherited metabolic defects. Typically it is not caused by dietary deficiency, even in vegetarians. Folic acid deficiency does not present often in adolescents; when it does, it typically is associated with malnutrition, unusual diets, malabsorption, or drugs that inhibit folate metabolism. Hematologic findings are nearly identical to those of pernicious anemia or other causes of vitamin B12 deficiency, but subacute combined degeneration of the spinal cord does not occur with folate deficiency. Pernicious anemia and folic acid deficiency can be differentiated by measuring serum levels of vitamin B12 and folate. Concentration of folate in red blood cells is a better long-term measure of folic acid sufficiency than serum folate concentrations. When pernicious anemia is suspected, the Schilling test should be performed. This identifies vitamin B12 malabsorption and whether that malabsorption is due to a deficiency of intrinsic factor. It is critical not to treat a person who has a vitamin B12 deficiency with folate because it will mask the neurologic disease while improving the hematologic picture. The issue is confused further by the possible reduction in red blood cell folate in vitamin B12 deficiency and potential decreased vitamin B12 levels in folate deficiency. A hematologist should be consulted to make the critical distinction between these two diagnoses. Macrocytosis can be a feature of Fanconi anemia, but this condition presents very rarely in adolescents, is not associated with nausea or canker sores, and usually is seen in conjunction with congenital malformations. Acquired aplastic anemia typically is not macrocytic. Methyltetrahydrofolate reductase deficiency is very rare and generally is associated with developmental delay.

References: Babior BM. The megaloblastic anemias. In: Beutler E, Lichtman MA, Coller BS, Kipps TJ, eds. Williams Hematology. 5th ed. New York, NY: McGraw-Hill, Inc; 1995:471-489 Whitehead VM, Rosenblatt DS, Cooper BA. Megaloblastic anemia. In: Nathan DG, Orkin SH, eds. Nathan and Oski's Hematology of Infancy and Childhood. 5th ed. Philadelphia, Pa: WB Saunders Co; 1998:385-422 Critique 57. Preferred Response: B

[View Question] Varicella-zoster immune globulin (VZIG) can ameliorate or prevent varicella in immunocompromised children if administered within 96 hours of exposure. The decision to provide VZIG is based on whether the exposed child is susceptible to varicella, is likely to develop severe disease, and has had sufficient exposure to result in infection. Determining the susceptibility to varicella may be complex. In healthy individuals, serologic testing accurately reflects immune status, but results may not be reliable in immunocompromised patients. Accordingly, a carefully obtained history of varicella is the primary consideration in determining susceptibility to infection. Children receiving chemotherapy constitute an important group of immunocompromised patients at risk for severe and even fatal varicella. They often have more and larger lesions, some of which can become hemorrhagic (Figure 57A), and they develop new lesions for longer periods. They also are at greater risk of developing complications such as pneumonitis. Occasional patients who have acute, rapidly evolving varicella associated with disseminated intravascular coagulation can die before antiviral therapy can be started. It is critical to determine whether the immunocompromised patients exposure was sufficient to result in infection and, therefore, necessitates administration of VZIG. The contagious state lasts from 1 to 2 days before to 5 days after the eruption of vesicles. Adequate exposure to a contagious individual includes anyone residing in the same household; nontransient face-to-face play (defined from as few as 5 minutes to as long as 1 hour by different experts); sharing a two- or four-bed room or adjacent beds in a larger ward in the hospital; and face-to-face contact with an infectious staff member, patient, or visitor. Infectious exposure to zoster requires intimate physical contact such as touching or hugging. Accordingly, the girl described in the vignette is a candidate for VZIG administration. The decision to continue or withhold chemotherapy after exposure and until the incubation period has ended is complex. It is complicated further by the potential of VZIG to prolong the normal incubation period from 21 days to up to 28 days. Any decision about chemotherapy should be based on the nature of the exposure, the patients clinical condition, and the intensity of the chemotherapeutic regimen. Patients who develop varicella should have their chemotherapy temporarily discontinued.

The use of varicella vaccine in patients receiving chemotherapy is under evaluation, although its use has been suggested in those who have acute lymphoblastic leukemia in remission and potentially in other lower risk patients. However, it is not used in patients who already have been exposed to varicella. Acyclovir should be administered to patients who develop varicella, but it is not given as prophylactic therapy. References: American Academy of Pediatrics. Varicella-zoster infections. In: Pickering LK, ed. 2000 Red Book: Report of the Committee on Infectious Diseases. 25th ed. Elk Grove Village, Ill: American Academy of Pediatrics; 2000:624-638 Gershon AA. Varicella-zoster virus. In: Feigin RD, Cherry JD, eds. Textbook of Pediatric Infectious Diseases. 4th ed. Philadelphia, Pa: WB Saunders Co; 1998:1769-1777 Rosenberg M. Passive immunization with varicella zoster immune globulin. Pediatr Rev. 1994;15:254 Critique 106. Preferred Response: C

[View Question] All children who have hemophilia and experience significant observed trauma should be treated prophylactically with the appropriate factor. The development of a significant hematoma also is an indication for treatment. Accordingly, the boy described in the vignette should receive an infusion of factor VIII. The management of even suspected intracranial hemorrhage requires immediate replacement of factor (factor VIII for hemophilia A or factor IX for hemophilia B) to a plasma level of 100%. If there is clinical or radiologic evidence of an intracranial hemorrhage, a hematologist who has expertise in managing hemophilia should be consulted for continuing therapy to maintain the factor at a safe level. The total duration of treatment is generally 10 to 14 days. Neurosurgical intervention may become necessary, but it is critical to provide factor replacement and adequate hemostasis to minimize the risks and maximize the benefits of surgery. Early intervention with factor may prevent the development of intracranial bleeding. Intracranial hemorrhage is the most dreaded complication of hemophilia, often causing severe neurologic deficits or death. This complication occurs in 2% to 8% of children who have the severe form of hemophilia. Approximately 33% of children who develop this complication die, and almost 50% of survivors have mental retardation, seizure disorders, or motor impairment. Antecedent trauma is recognized in only 50% of cases, probably because the trauma may be relatively minor and forgotten or because there often is a delay of days and even weeks from the time of injury to the development of symptoms. With younger children, the trauma simply may not have been observed by the parents. One of the more difficult problems in treating hemophilia is managing the child who has had trauma but exhibits no signs of intracranial bleeding. The ideal approach is to treat

such children before intracranial bleeding begins or becomes extensive. Unfortunately, it can be difficult (particularly with very young children), to determine the extent of trauma and whether it justifies prophylactic infusions of coagulation factor. However, judgement always should lean toward intervention. Although it may not be practical to study every child radiologically, any child who has lost consciousness, shown any signs of concussion such as headache, or has had concerning degree of trauma should be treated immediately with prophylactic factor and then undergo computed tomography (CT) of the head. Intracranial bleeding should be identified readily on CT, even without the use of contrast. Desmopressin plays a role in the treatment of patients who have mild hemophilia A whose factor VIII levels are generally greater than 10%. It is not appropriate therapy for patients who have more severe disease, with factor VIII levels of less than 10%. References: Bell B, Canty D, Audet M. Hemophilia: an updated review. Pediatr Rev. 1995;16:290-298 DiMichele D. Hemophilia 1996. New approach to an old disease. Pediatr Clin North Am. 1996;43:709-736 Montgomery RR, Gill JC, Scott JP. Hemophilia and von Willebrand disease. In: Nathan DG, Orkin SH, eds. Nathan and Oski's Hematology of Infancy and Childhood. 5th ed. Philadelphia, Pa: WB Saunders Co; 1998:1631-1659 Critique 152. Preferred Response: B

[View Question] Approximately 20% of children who have Down syndrome develop either acute lymphoblastic or acute myeloblastic leukemia. The proportion of these two forms of leukemia is similar to that seen in the general population, but the prognosis differs. Children who have acute lymphoblastic leukemia and Down syndrome have a worse prognosis than that of otherwise normal children. In contrast, those who have acute myeloblastic (usually megakaryoblastic) leukemia and Down syndrome have a better prognosis than that of otherwise normal children. One third of infants who have Down syndrome develop transient abnormal myelopoiesis, which has been described as transient leukemia. The mechanism of this transient abnormality remains unknown. It usually occurs in neonates and may be an incidental finding on a complete blood count or may be recognized because of hepatosplenomegaly. It typically resolves spontaneously in the first 3 months of life and usually requires no specific therapy. Most affected infants have the phenotypic features of Down syndrome, but it can occur in phenotypically normal children who have a mosaicism for trisomy 21. Up to 30% of children who have transient abnormal myelopoiesis will develop acute myeloblastic leukemia in the first 3 years of life. Thus, this transient disorder

may represent a preleukemic process, but no initial clinical or laboratory indicators have been found to predict the children in whom leukemia will develop. A few children who have Down syndrome have developed aplastic anemia, but it is not known whether this is a true association or a coincidence. Biochemical and functional abnormalities of platelets have been described in children who have Down syndrome, but they have not been associated with clinical bleeding. Macrocytosis can be associated with Down syndrome, but anemia is not. Thrombocytosis is not expected in children who have Down syndrome. References: Freedman MH, Doyle JJ. Inherited bone marrow failure syndromes. In: Lilleyman JS, Hann IM, Blanchette VS, eds. Pediatric Hematology. 2nd ed. London, England: Churchill Livingstone; 1999:23-49 Sato Y, Rowley JD. Chromosomal abnormalities in childhood hematologic malignant diseases. In: Nathan DG, Orkin SH, eds. Nathan and Oski's Hematology of Infancy and Childhood. 5th ed. Philadelphia, Pa: WB Saunders Co; 1998:1147-1182 Critique 200. Preferred Response: A

[View Question] Accurate determination of the absolute neutrophil count (ANC) is a critical test among immunocompromised children such as the boy described in the vignette because of the important role that neutrophils play in controlling bacterial and fungal infections. The ANC traditionally has been determined by measuring the percentage of segmented neutrophils and band forms and multiplying this by the total white blood cell count. Automated counters now employ the white blood cell count and an automated differential to provide an ANC. When evaluating a febrile child receiving chemotherapy, the ANC is critical for determining both the urgency with which antibiotic therapy is needed and the spectrum of antibiotics indicated. A blood culture also should be obtained to determine if sepsis is present. A urine culture should be obtained, but it is less critical because the urinary tract is a less common site of infection. Chest radiographs are not usually helpful in the absence of a specific clinical indication. Lumbar puncture is indicated only if clinical signs of meningitis are present. The need for other studies, such as liver function tests, depends on the specific patients history and chemotherapeutic regimen. The degree of neutropenia helps to determine the risk of infection among immunocompromised patients. There is some normal variation in ANC with age (lower limits of 1.0 x 109/L [1,000/mm3] from 2 weeks to 6 months of age with a rise to 1.5 to 109/L [1,500/mm3] from 1 to 8 years and an increase to 1.8 x 109/L [1,800/mm3] at about age 10 and thereafter) and with race (ANCs approximately 0.5 x 109/L [500/mm3] lower in AfricanAmericans). Regardless of the normal limits, there appears to be only a modest increase in the risk of infection when the ANC decreases to 0.5 to 1.0 x 109/L (500 to 1,000/mm3). Once

it falls below 0.5 x 109/L (500/mm3), however, the risk of serious infection substantially increases as the count falls further. Neutropenia occurs most often in association with routine viral illnesses in otherwise healthy children, who rarely develop serious infections. However, children who have severe neutropenia due to drugs or congenital neutropenia are more likely to become seriously ill. Severe neutropenia with infectious complications occurs most commonly among children receiving chemotherapy. They are at risk not only because of the potentially profound neutropenia, but because of the immunosuppressive effects of therapy, the mucositis that often occurs concomitantly, and the frequent use of indwelling central venous catheters. These children require particularly aggressive antibiotic management of fever, which involves the use of broad-spectrum intravenous antibiotics in the hospital at least initially. Broad-spectrum oral antibiotic use is being evaluated in adults, but its efficacy and safety remain to be proven in children. The safety of oral antibiotics in an ambulatory setting is unproven in either adults or children.

References: Boxer LA, Blackwood RA. Leukocyte disorders: quantitative and qualitative disorders of the neutrophil, part 1. Pediatr Rev. 1996;17:19-28 Pizzo PA, Mueller BU. Infectious complications in children with hematologic disorders. In: Nathan DG, Orkin SH, eds. Nathan and Oski's Hematology of Infancy and Childhood. 5th ed. Philadelphia, Pa: WB Saunders Co; 1998:1738-1759 Ryan DH. Automated analysis of blood cells. In: Hoffman R, Benz EJ Jr, Shattil SJ, et al, eds. Hematology: Basic Principles and Practice. 3rd ed. New York, NY: Churchill Livingstone; 2000:2469-2480 Critique 243. Preferred Response: E

[View Question] Vitamin B12 deficiency can result from decreased intake or absorption of the vitamin. Pernicious anemia is an important cause of vitamin B12 deficiency, but it is rare in childhood. In this disorder, autoimmune gastritis causes achlorhydria and reduced or absent intrinsic factor, which is essential for the absorption of vitamin B12. Unrecognized pernicious anemia in the mother can cause vitamin deficiency in an infant, but this rarely occurs because untreated pernicious anemia often causes infertility. More often, vitamin B12 deficiency in infants is due to simple dietary deficiency of the vitamin in the mother. The infant becomes deficient not only because of reduced placental transfer of cobalamin, but because of deficient vitamin B12 in human milk. Most often the mother is a vegan who does not eat meat, fish, eggs, or cheese, as noted for the mother described in the vignette. This is particularly common among immigrants to the West where improved hygiene in food preparation and handling minimize the bacterial and

fungal content of food, which is likely the source of vitamin B12 in developing countries. Some food faddists also can consume a diet deficient in vitamin B12. Less common causes of maternal or childhood vitamin B12 deficiency include gastric resection (resulting in loss of the mucosa that produces intrinsic factor), fish tapeworms, bacterial overgrowth of the small intestine, and bowel disease affecting the terminal ileum where the vitamin is absorbed (eg, Crohn disease, ulcerative colitis, surgical resection of the terminal ileum). Infants who have vitamin B12 deficiency may present with vomiting, anorexia, diarrhea, and failure to thrive. Laboratory studies reveal macrocytic anemia and in severe cases, leukopenia and thrombocytopenia. A peripheral blood smear (Figure 243A) shows large oval macrocytes and hypersegmented neutrophils. Folate deficiency can present with similar symptoms and hematologic changes to vitamin B12 deficiency. It often is associated with malnutrition, but it also occurs in children who have hemolytic anemias, human immunodeficiency virus infection, hepatitis, or malaria. Infants exclusively fed goat milk, which contains very little folate, are at high risk for this deficiency. Malabsorptive disorders, particularly celiac disease and extensive Crohn disease, also are associated with folate deficiency. Unlike vitamin B12 deficiency, folate deficiency is not associated with permanent neurologic damage. A number of rare inherited metabolic disorders can result in poor absorption, utilization, and transport of both folate and vitamin B12, including methylmalonic aciduria and intrinsic factor deficiency. The presence of macrocytosis differentiates early mild macrocytic anemia from normocytic physiologic anemia of infancy. References: Babior BM. The megaloblastic anemias. In: Beutler E, Lichtman MA, Coller BS, Kipps TJ, eds. Williams Hematology. 5th ed. New York, NY: McGraw-Hill, Inc; 1995:471-489 Rosenblatt DS, Hoffbrand AV. Megaloblastic anemia and disorders of cobalamin and folate metabolism. In: Lilleyman JS, Hann IM, Blanchette VS, eds. Pediatric Hematology. 2nd ed. London, England: Churchill Livingstone; 1999:167-184 Whitehead VM, Rosenblatt DS, Cooper BA. Megaloblastic anemia. In: Nathan DG, Orkin SH, eds. Nathan and Oski's Hematology of Infancy and Childhood. 5th ed. Philadelphia, Pa: WB Saunders Co; 1998:385-422 2000 Self-Assessment Exercise XV. Disorders of the blood and neoplastic disorders [Return to Category List] Questions [Print Directions]

Question 11.

Answer.

A 4-year-old boy develops massive bleeding following a tonsillectomy. A transfusion is indicated, but his parents are extremely concerned about the risk of a transfusion-mediated infection. They want to know what tests are performed on donated units of blood before they consent to the procedure. Of the following, your discussion is MOST likely to include the statement that A. B. C. all units are tested only for hepatitis B and C all units are tested only for human immuno-deficiency virus (HIV) all units are tested for HIV, hepatitis B, and hepatitis C

D. all units are tested for HIV, hepatitis B, hepatitis C, cytomegalovirus, and EpsteinBarr virus E. HIV only units obtained from donors who have one or more risk factors are screened for

Question 45.

Answer.

A 3-year-old girl recovering from a viral illness is pale and jaundiced. Findings include: splenomegaly; hemoglobin, 5.9 g/dL; mean corpuscular volume, 92 fL; mean corpuscular hemoglobin concentration, 37 g/dL (normal, 32.1 to 36.5 g/dL); white blood cell count, 13,500/mm3, with a normal differential count; platelet count, 431,000/mm3; and reticulocyte count, 34%. Her father and paternal uncles underwent cholecystectomies in their 20s. Of the following, the MOST likely diagnosis is A. B. C. D. E. Blackfan-Diamond syndrome glucose-6-phosphate dehydrogenase deficiency hereditary elliptocytosis hereditary spherocytosis pyruvate kinase deficiency Answer.

Question 78.

A 5-year-old boy has been diagnosed as having autoimmune hemolytic anemia with a warm antibody. He has been receiving high-dose corticosteroids for the past 24 hours, but his hemoglobin concentration has continued to fall (6.6 g/dL to 4.8 g/dL). Cross-matching procedures fail to identify any compatible units of red blood cells. Of the following, the most appropriate NEXT step in the management of this patient is to A. administer intravenous gamma globulin

B. C. D. E.

initiate cytotoxic therapy obtain a surgical consultation for splenectomy perform an exchange transfusion transfuse with the least incompatible blood available

Question 114. Answer. A 6-year-old girl complains of pain in her legs. Physical examination reveals petechiae, purpura, and marked pallor. Her spleen is palpable 3 cm below the left costal margin. Laboratory findings include: hemoglobin, 6.3 g/dL; white blood cell count, 2,400/mm3; and platelets, 22,000/mm3. Of the following, the MOST likely diagnosis is A. B. C. D. E. acute lymphoblastic leukemia aplastic anemia autoimmune hemolytic anemia Henoch-Schnlein purpura immune thrombocytopenic purpura

Question 151. Answer. A 4-year-old girl develops pallor and icterus 1 week after having an upper respiratory tract infection. A complete blood count measured last year was normal. There is no family history of anemia. Findings include: a palpable spleen; hemoglobin, 4.9 g/dL; reticulocyte count, 31%; indirect bilirubin, 3.9 mg/dL; and normal red blood cell indices, white blood cell count, differential count, and platelet count. The laboratory study that is MOST helpful to confirm the cause of these findings is A. B. C. D. E. bone marrow examination direct Coombs test glucose-6-phosphate dehydrogenase assay hemoglobin electrophoresis osmotic fragility test

Question 188. Answer. An 8-year-old child is brought to you because of leg pain. The pain is greatest at night and is relieved by ibuprofen. There is some local tenderness over the thigh and slight atrophy of

the affected limb. A radiograph of the femur reveals a radiolucent nidus surrounded by sclerotic bone, and a bone scan shows increased uptake in that area. The MOST likely explanation for these findings is A. B. C. D. E. Ewing sarcoma hairline fracture osteochondroma osteogenic sarcoma osteoid osteoma

Question 219. Answer. A 3-year-old girl suddenly develops a fever of 40C (104F), a stiff neck, petechiae, and purpura. Laboratory findings include a hemoglobin of 9.1 g/dL and a platelet count of 14,000/mm3. Of the following, the MOST appropriate next hematologic tests to obtain are A. B. C. D. E. bleeding time and factor VIII assay factor VIII and IX assays fibrinogen and fibrin split products platelet-associated antibody and Coombs test prothrombin and partial thromboplastin times

Question 250. Answer. A 10-month-old girl has had eight episodes of fever, mouth sores, and irritability over the past 6 months that last several days and resolve spontaneously. Physical examination reveals ulcers in the oral mucosa, gingivitis, and cervical adenopathy. Results of laboratory evaluation include: white blood cell count, 3,000/mm3; 3% neutrophils, 84% lymphocytes, and 13% monocytes on differential count; hemoglobin, 11.0 g/dL; and platelet count, 334,000/mm3. The MOST important test(s) to obtain is (are) A. B. C. D. E. bone marrow examination Epstein-Barr virus titers serial absolute neutrophil counts serum B12 concentration serum folate concentration

Answers Critique 11 Preferred Response: C

[View Question] The safety of the blood supply is a critical concern, as exemplified by the initially unrecognized transmission of human immunodeficiency virus (HIV) via transfusions. All units of blood donated in the United States for transfusion today must be screened for antibodies to HIV (anti-HIV), human immunodeficiency virus antigen (HIV-1-Ag), human T-cell lymphotrophic virus (anti-HTLV), hepatitis B core antigen (anti-HBc), hepatitis B surface antigen (HBsAg), hepatitis C virus (anti-HCV), and syphilis. The reagents and techniques used must be licensed by the United States Food and Drug Administration. These tests are required by the American Association of Blood Banks. Unfortunately, these screening tests cannot eliminate the risk of transmission totally because a recently infected donor may not have developed detectable antigen or antibody. Therefore, additional precautions are taken. The sole use of unpaid volunteer donors decreases the risk factors for many infections and more importantly, tends to increase the reliability of historical information obtained from donors. All donors receive detailed educational materials informing them of the clinical signs and symptoms associated with HIV infection and acquired immunodeficiency syndrome (AIDS), of high-risk activities for transmission of AIDS, and of the importance of refraining from donating blood if they have engaged in these activities or have experienced specific signs and symptoms. The volunteering of this information truthfully and accurately during the health assessment of the donor is critical to the successful exclusion of those whose blood may transmit disease to the recipient. A qualified individual then questions donors and determines whether the responses are satisfactory. In addition to questions specifically directed toward HIV infection, donors are asked about risk or exposure to hepatitis, gonorrhea, malaria, babesiosis, Brucella, parvovirus, Toxoplasma, and certain trypanosomes. Donors also are questioned regarding illicit use of intravenous drugs, having received blood in the prior 12 months, and having been deferred previously as a donor. A limited physical examination of the donor also is performed, including assessment of general appearance, examination for skin lesions and signs of intravenous drug abuse on both arms, and measurement of weight, temperature, pulse, blood pressure, and hemoglobin. The results of this examination must be noted. Although obtaining this history and performing this examination do not eliminate the risk of infections in the donor, they do reduce it. Testing for evidence of past cytomegalovirus (CMV) infection is not performed routinely. Up to 70% of donors may have anti-CMV antibodies, but this poses minimal risk to the usual recipient. However, low-birthweight preterm infants born to CMV-seronegative mothers and immunocompromised, CMV-seronegative individuals would be affected adversely by CMV infection and will reduce their risk of CMV transmission greatly by receiving only blood components that have been documented as serologically negative for CMV. CMV testing is performed only when trying to identify donor units specifically for such recipients. Testing for Epstein-Barr virus is not performed routinely.

References: Circular of Information for the Use of Human Blood and Blood Components. American Association of Blood Banks, America's Blood Centers, American Red Cross. 1998 McCullough J. Blood procurement and screening. In: Beutler E, Lichtman MA, Coller BS, Kipps TJ, eds. Williams Hematology. 5th ed. New York, NY: McGraw-Hill, Inc; 1995:1618-1621 Critique 45 Preferred Response: D

[View Question] Presenting symptoms and signs of hereditary spherocytosis include pallor, icterus, splenomegaly, or symptomatic gallbladder disease. In some patients, the first sign of the disorder may be anemia identified on hemoglobin screening as part of a health supervision visit. Although the diagnosis usually is recognized later in life, it may present in the neonate. Approximately 50% of affected patients have exaggerated neonatal hyperbilirubinemia, which may lead to the diagnosis. The anemia often is more severe in the first weeks of life, and pallor and poor feeding also may lead to the diagnosis. Transfusions are required more frequently during this period. This presentation reflects the combination of the membrane defect causing hemolysis and the shortened erythrocyte survival among healthy neonates. Children who have hereditary spherocytosis often present acutely with severe anemia when the balance of their accelerated red blood cell destruction no longer is compensated by enhanced production of erythrocytes. This typically is exemplified by the aplastic crisis in which a viral infection (most often parvovirus B 19) suppresses erythropoiesis while hemolysis continues. Reticulocyte counts fall and may reach 0. Hemoglobin concentrations may decrease to life-threatening levels within several days. Any pre-existing jaundice actually may decrease, as a decreasing mass of hemoglobin generates less bilirubin as a byproduct of red blood cell breakdown. Thrombocytopenia also may be noted. An acute viral illness also may create a more complex picture in affected children. In addition to inhibition of erythropoiesis, there may be a sudden enlargement of the spleen due to the infection and sequestration of spherocytic erythrocytes. The combination of inhibited erythropoiesis, enhanced splenic sequestration, and destruction of red blood cells may cause a dramatic reduction in hemoglobin. Clinically, increasing pallor and fatigue, jaundice, and an enlarging spleen are noted, as reported for the girl in the vignette. The diagnosis of hereditary spherocytosis should be suspected in a child who presents with hemolytic anemia, an elevated mean corpuscular hemoglobin concentration (MCHC), and the classic finding of spherocytes (small, dense erythrocytes lacking central pallor). Because the disorder is inherited as an autosomal dominant trait, there may be a family history of anemia, jaundice, cholelithiasis, cholecystectomy, or splenectomy. The osmotic fragility test remains the primary diagnostic study. The clinical presentation of hereditary elliptocytosis may be very similar to that of hereditary spherocytosis, but elliptocytes are the predominant finding on the blood smear and MCHC is normal. Neonates who have mild hereditary elliptocytosis may present with

infantile poikilocytosis; in contrast to the predominant elliptocytosis seen later in life, red blood cell budding, fragmentation, and marked variation in cell shape (poikilocytosis) occur. Glucose-6-phosphate dehydrogenase deficiency is a sex-linked recessive disorder that does not present with an elevated MCHC. Pyruvate kinase deficiency, which is much less common than hereditary spherocytosis, presents with fewer abnormalities on blood smear and a normal MCHC. Children who have Blackfan-Diamond syndrome generally present with anemia in the first year of life, and the condition is associated with aplasia of the red blood cells, not hemolysis. The reticulocyte count will be very low and the MCHC will be normal. References: Bestak M. The aplastic crisis in hereditary spherocytosis. Pediatr Rev. 1994;15:212 Gallagher PG, Forget BC, Lux SE. Disorders of the erythrocyte membrane. In: Nathan DG, Orkin SH, eds. Nathan and Oski's Hematology of Infancy and Childhood. 5th ed. Philadelphia, Pa: WB Saunders Co; 1998:578-601 Koch WC, Massey GV. Aplastic crisis. Pediatr Rev. 1990; 12:142-148 Mustafa MM, McClain KL. Diverse hematologic effects of parvovirus B19 infection. Pediatr Clin North Am. 1996;43: 809-821 Critique 78 Preferred Response: E

[View Question] Erythrocyte transfusions may be lifesaving in children who have severe autoimmune hemolytic anemia. Although corticosteroids remain the mainstay of therapy in children who have warm antibody-mediated autoimmune hemolytic anemia, those who develop cardiovascular compromise or whose hemoglobin falls to less than 5 g/dL should receive transfusions of red blood cells. The goal of transfusion is to increase the hemoglobin to 6 to 8 g/dL. The initial challenge is identification of compatible blood. The autoantibody in these patients often reacts with all normal red blood cells, making it impossible to identify compatible blood. Therefore, it is important to identify the patient's ABO and Rh type and transfuse with the least incompatible units of blood available. Strong antibodies directed against the patient's red blood cells may mask the presence of antibodies against donor red blood cells in those who have been transfused previously. A second challenge is limiting the risk of transfusion reactions, although these are not common. In reality, the transfused cells usually are destroyed no faster than the patient's own antibody-coated erythrocytes. Even though they may be destroyed quickly, the transfused cells can provide a margin of safety that may be critical in one of the most severe and life-threatening hemolytic anemias of childhood. Because occasional patients develop severe acute hemolysis with hemoglobinemia, hemoglobinuria, and acute renal

failure, transfusions should be administered slowly and patients monitored closely for intravascular hemolysis. An additional challenge is managing the patient who has a cold-reactive antibody rather than the more typical warm-reactive, immunoglobulin G (IgG) antibody. A blood warmer must be used to keep the temperature of the blood close to body temperature, and the patient and the room should be kept warm. If the patient becomes cold, the cold antibody may attach to and hemolyze more red blood cells in vivo. Other therapeutic modalities may be used in the treatment of those who have warm-mediated autoimmune hemolytic anemia. The administration of intravenous gamma globulin is helpful in approximately one third of patients. Splenectomy and cytotoxic therapy usually is limited to patients who have more severe and chronic disease. Exchange transfusion has been used in these patients, but it should be limited to those whose condition fails to stabilize with corticosteroids and transfusions of red blood cells. References: Thomas AT. Autoimmune hemolytic anemias. In: Lee GR, Foerster J, Lukens J, Paraskevas F, Greer JP, Rodgers GM, eds. Wintrobe's Clinical Hematology. 10th ed. Baltimore, Md: Williams & Wilkins; 1999:1233-1263 Ware RE, Rosse WF. Autoimmune hemolytic anemia. In: Nathan DG, Orkin SH, eds. Nathan and Oski's Hematology of Infancy and Childhood. 5th ed. Philadelphia, Pa: WB Saunders Co; 1998:499-522 Critique 114 Preferred Response: A

[View Question] The combination of anemia, thrombocytopenia, and leukopenia (or neutropenia even if the total white blood cell count is normal) may be due to a variety of serious illnesses, but the possibility of acute leukemia or other infiltrative disorder of bone marrow must be considered. Most children who have acute leukemia exhibit pancytopenia, although the anemia and thrombocytopenia may be mild. Fever, bleeding, bone pain, lymphadenopathy, hepatomegaly, and splenomegaly are common. Blasts frequently are noted in the peripheral blood, but they may be absent in children who have low or normal white blood cell counts (although most of these children have neutropenia). Metabolic abnormalities such as hyperuricemia, hyperphosphatemia, and elevated lactate dehydrogenase strongly suggest an underlying malignancy. Viral illnesses, such as Epstein-Barr or cytomegalovirus infection, also can produce fever, lymphadenopathy, hepatosplenomegaly, and pancytopenia. Although the atypical lymphocytosis associated with these illnesses usually can be distinguished from the blasts of leukemia, a bone marrow examination may be necessary to rule out malignancy. Examining the marrow also will help diagnose the less common possibility of aplastic anemia. Prompt diagnosis is critical in the severe, acquired form of aplastic anemia because early bone

marrow transplantation should be considered. Children who have aplastic anemia generally do not have bone pain, lymphadenopathy, hepatosplenomegaly, or metabolic abnormalities. Several other disorders must be considered in the child who has pancytopenia. Sick, acutely septic children may present with leukopenia as a result of sepsis as well as anemia and thrombocytopenia secondary to disseminated intravascular coagulation. Megaloblastic anemias (with macrocytosis and hypersegmented neutrophils) and myelodysplastic and preleukemic syndromes also must be considered in the differential diagnosis. The typical clinical presentation of involvement of more than one marrow cell line is cutaneous bleeding (purpura and petechiae) due to thrombocytopenia and pallor due to anemia; leukopenia or neutropenia less frequently are evident. Although the previously noted disorders may present initially with only anemia and thrombocytopenia, the possibility of immune thrombocytopenia with anemia as a result of bleeding, the combination of immune thrombocytopenia and autoimmune hemolytic anemia (Evan syndrome), and the microangiopathic hemolytic anemias (eg, hemolytic-uremic syndrome, thrombotic thrombocytopenic purpura) also must be considered. Vascular disorders, such as Henoch-Schnlein purpura, present with purpura in the absence of thrombocytopenia, but all blood counts should be normal. References: Berliner N, Duffy TP, Abelson HT. Approach to the adult and child with anemia. In: Hoffman R, Benz EJ Jr, Shattil SJ, Furie B, Cohen HJ, Silberstein LE, eds. Hematology: Basic Principles and Practice. 2nd ed. New York, NY: Churchill Livingstone; 1995:468-483 Sills RH. Indications for bone marrow examination. Pediatr Rev. 1995;16:226-228 Critique 151 Preferred Response: B

[View Question] The evaluation of children who have acute anemia should include a direct Coombs (direct antiglobulin) test to exclude the possibility of autoimmune hemolytic anemia. This disorder typically occurs in a previously well child who presents with pallor, fatigue, and icterus during or soon after a viral illness, such as the girl described in the vignette. Hemoglobinuria may result from intravascular hemolysis. Physical examination reveals pallor, scleral icterus, and hepatosplenomegaly, although the organomegaly rarely is massive. Hemoglobin concentrations of 4 to 7 g/dL often are noted in patients who have autoimmune hemolytic anemia, and the anemia usually is normocytic. A high mean corpuscular volume may result from a reticulocytosis or antibody-induced agglutination of red blood cells, which causes an artifactual macrocytosis that is detected by electronic counters as one large single cell. White blood cell and platelet counts typically are normal or modestly increased. The differential count also usually is normal. The blood smear reveals small spherocytes that are the result of the typical immunoglobulin G (IgG) warm antibody that mediates this disorder. Polychromasia is common because of the reticulocytosis, although reticulocytopenia may occur in some patients.

The direct Coombs test identifies antibody or complement on the surface of circulating erythrocytes. After erythrocytes are washed to remove plasma proteins, they are mixed with a rabbit polyclonal antiserum that binds both human gamma globulin and complement (usually C3). A positive Coombs test is strongly suggestive of autoimmune hemolytic anemia, although on rare occasions the disorder is associated with a negative study. The disease may be mediated by an IgG (warm) antibody or IgM (cold) antibody. The clinical features and treatment vary, depending on the type of antibody involved. IgGmediated hemolysis is most common in children. None of the other differential diagnostic possibilities of acute anemia demonstrates a positive Coombs test. The osmotic fragility test is used to diagnose hereditary spherocytosis. This disorder also presents with spherocytes, but these cells are more predominant in the blood smear, and the mean corpuscular hemoglobin concentration usually is elevated. The glucose-6-phosphate dehydrogenase (G6PD) assay is performed to exclude G6PD deficiency, a sex-linked recessive disorder in which spherocytes may be noted but are less frequent than in autoimmune hemolytic anemia. Hemoglobin electrophoresis generally excludes hemoglobinopathies, but this test usually is performed as part of neonatal screening programs (mandatory in most states) and, therefore, probably already had been done for the girl described in the vignette. The elevated reticulocyte count and serum bilirubin confirm a hemolytic disorder, excluding the need to evaluate bone marrow function. References: Thomas AT. Autoimmune hemolytic anemias. In: Lee GR, Foerster J, Lukens J, Paraskevas F, Greer JP, Rodgers GM, eds. Wintrobe's Clinical Hematology. 10th ed. Baltimore, Md: Williams & Wilkins; 1999:1233-1263 Ware RE, Rosse WF. Autoimmune hemolytic anemia. In: Nathan DG, Orkin SH, eds. Nathan and Oski's Hematology of Infancy and Childhood. 5th ed. Philadelphia, Pa: WB Saunders Co; 1998:499-522 Critique 188 Preferred Response: E

[View Question] Osteoid osteoma is a common benign bony tumor of childhood. It typically presents with sharp, boring pain that often is unrelenting, worse at night, and almost without exception completely relieved by aspirin. With the dramatic decrease in the use of aspirin among children, naproxen has been found to duplicate the relief provided by aspirin and is more effective than ibuprofen. Acetaminophen is relatively ineffective. Osteoid osteoma occurs more often among males and typically presents from 5 to 24 years of age. It can occur in any bone, but involves the femur or tibia in 50% of patients. The spine is another fairly common site, and scoliosis may complicate a lesion in this location. Spinal lesions may be particularly difficult to localize with routine radiography. Tenderness at the site of the osteoid osteoma and limp or atrophy of the involved limb may

be noted. Accordingly, osteoid osteoma is the most likely diagnosis for the child described in the vignette. On plain radiographs, the tumor typically produces a radiolucent nidus with surrounding reactive bone. In some patients, the nidus can be visualized only by computed tomography with 1 to 2 mm sections through the area of a suspected lesion. Pain may be present before a lesion can be demonstrated radiologically; in these patients, a bone scan is helpful in locating the lesion. Although the lesion and this pain will resolve eventually, surgical excision of the nidus offers dramatic relief for patients who have persistent pain. A hairline fracture may be difficult to identify radiologically, but the history and pattern of pain should lead to the appropriate diagnosis. Osteochondromas are the most common benign tumors in children. They can be differentiated from osteoid osteomas by their presentation as a painless, hard, and nontender mass that usually is located in the metaphyses of long bones. Radiographs demonstrate a bony outgrowth from the cortex. Simple bone cysts are very common among children, presenting with pain that usually is associated with a pathologic fracture from expansion and weakening of the metaphysis. On radiography, there is a large lucency that may be traversed with regions of cortical bone. The most common primary malignant bone tumors are osteogenic sarcoma and Ewing sarcoma. Both typically present with pain and the development of a mass. Routine radiographs in osteogenic sarcoma demonstrate a fairly typical pattern of both bone destruction and production as well as periosteal elevation. In Ewing sarcoma, the lesion appears primarily lytic and destructive, with multilamellar periosteal reaction (onion skin appearance). References: Skinner SR. Orthopedic problems in childhood. In: Rudolph AM, ed. Rudolph's Pediatrics. 20th ed. Stamford, Conn: Appleton & Lange; 1996:2151-2153 Springfield DS. Bone and soft tissue tumors. In: Morrissy RT, Weinstein SL, eds. Lovell and Winter's Pediatric Orthopedics. 4th ed. Philadelphia, Pa: Lippincott-Raven Publishers; 1996:433-435 Critique 219 Preferred Response: E

[View Question] Disseminated intravascular coagulation (DIC) is a condition that usually is associated with shock and results in uncontrolled activation of the hemostatic system. Although the initial pathology involves microvascular thrombosis, the predominant process becomes hemorrhagic as the body is unable to generate coagulation factors and platelets quickly enough to compensate for their overuse. The most common etiology of DIC in children is bacterial sepsis. Meningococcus is the most frequent pathogen, but Haemophilus sp, Salmonella sp, group B Streptococcus, S pneumoniae, and rickettsiae are other causative organisms. The girl described in the

vignette has a stiff neck, high fever, and purpura, which suggests a diagnosis of DIC and bacterial meningitis. DIC also is triggered by hypotensive shock, tissue injuries (eg, massive burns, head trauma, crush injury, profound hypoxia, hyperthermia, hypothermia), and toxins (eg, snake bites). Chronic DIC is much less common in children than in adults and usually is related to underlying malignancies, such as acute promyelocytic leukemia. No single study can establish the diagnosis of DIC, but the combination of thrombocytopenia and prolonged prothrombin time (PT) and partial thromboplastin time (PTT) in a profoundly ill child who has no prior history of bleeding difficulties is highly suggestive of the disorder. Accordingly, it is most appropriate to measure both PT and PTT in the child described in the vignette. Other frequent findings of DIC include microangiopathic changes of red blood cell fragmentation; reductions in factors V, VIII, fibrinogen, and prothrombin; and elevations in fibrin split products. However, the results of these additional studies vary among affected patients and are not essential to establish the diagnosis in what usually is a very acute situation. Measuring the bleeding time is not helpful because the thrombocytopenia usually will prolong it. DIC is not an autoimmune phenomenon, so studies such as plateletassociated antibody and the Coombs test are not indicated. Successful treatment of DIC requires management of the primary underlying disease; death usually is due to the underlying disease, not the coagulopathy. Replacement of clotting factors and platelets is only a temporizing measure. In the absence of clinical bleeding, the coagulopathy typically does not require treatment. In children who experience clinical bleeding, platelet transfusions and plasma products can improve hemostasis until the underlying disorder is controlled, thereby resolving the DIC. References: Andrew M, Montgomery RR. Acquired disorders of hemostasis. In: Nathan DG, Orkin SH, eds. Nathan and Oski's Hematology of Infancy and Childhood. 5th ed. Philadelphia, Pa: WB Saunders Co; 1998:1682-1683 Grosset ABM, Rodgers GM. Acquired coagulation disorders. In: Lee GR, Foerster J, Lukens J, Paraskevas F, Greer JP, Rodgers GM, eds. Wintrobe's Clinical Hematology. 10th ed. Baltimore, Md: Williams & Wilkins; 1999:1739-1753 Murphy S, Nepo A, Sills R. Consultation with the specialist: thrombocytopenia. Pediatr Rev. 1999;20:64-69 Critique 250 Preferred Response: C

[View Question] Cyclic neutropenia is characterized by regular, periodic oscillations in the number of polymorphonuclear neutrophils in the peripheral blood. This rare granulopoietic disorder is presumed to be a regulatory defect involving early hematopoietic precursor cells. In approximately 25% of affected patients there is an autosomal dominant inheritance.

The mean oscillatory period is 213 days during which the absolute neutrophil count reaches a nadir of 0 and remains below 200 cells/mm3 for at least 3 to 5 days. Even during the peak of the cycle, neutrophil counts often remain below 1,900 cells/mm3. Cycling of reticulocytes, platelets, and other leukocytes often occurs, leading some to use the term cyclic hematopoiesis. Bone marrow aspirates obtained during such periods of neutropenia demonstrate myeloid hypoplasia or an apparent arrest of myeloid development at the myelocyte stage. Clinical symptoms also occur during the neutropenic phase of the cycle, with most patients suffering malaise, fever, gingivitis, oral ulcers, pharyngitis, and associated lymphadenopathy. More serious complications, including pneumonia, mastoiditis, recurrent vaginal or rectal mucosal ulcers, and Clostridium perfringens sepsis, occasionally occur. Improvement may be seen with age, and in these individuals, the cycles become less dramatic and resemble those seen in patients who have chronic neutropenia. Although often believed to be a benign condition, 10% of patients who have this disorder die of infectious complications. The diagnosis of cyclic neutropenia should be considered in children who have a clinical history of recurrent fever, gingivitis, and mouth ulcers, such as the girl described in the vignette. Absolute neutrophil counts should be obtained two to three times weekly for 6 to 8 weeks to establish whether a 21 day cycle can be identified. Management of cyclic neutropenia requires prompt identification and treatment of infections, including careful attention to oral and dental hygiene. Prophylactic administration of granulocyte colony stimulating factor also appears to be beneficial. Unlike the severe, noncyclic congenital neutropenias, cyclic neutropenia has not been associated with an increased risk of developing myeloid leukemia. Because cyclic neutropenia is rare, other etiologies of neutropenia must be considered. Viral infections are the most common etiology, but affected patients rarely develop clinical infections and have no history of cyclic symptoms. Epstein-Barr virus infections often are associated with fever, tonsillopharyngitis, and cervical adenopathy, but mucosal ulcers and gingivitis are not characteristic. A bone marrow examination may show a decrease in neutrophil precursors, but this evaluation is not necessary to establish the diagnosis. Megaloblastic anemia, due to either B12 or folate deficiency, causes neutropenia, but rarely is the neutropenia severe, and usually it is associated with hypersegmentation of the neutrophils, macrocytic anemia, and thrombocytopenia. References: Boxer LA, Blackwood RA. Leukocyte disorders: quantitative and qualitative disorders of neutrophil, Part 1. Pediatr Rev. 1996;17:19-28 Dinauer MC. The phagocyte system and disorders of granulopoiesis and granulocyte function. In: Nathan DG, Orkin SH, eds. Nathan and Oski's Hematology of Infancy and Childhood. 5th ed. Philadelphia, Pa: WB Saunders Co; 1998:910-925

Watts RG. Neutropenia. In: Lee GR, Foerster J, Lukens J, Paraskevas F, Greer JP, Rodgers GM, eds. Wintrobe's Clinical Hematology. 10th ed. Baltimore, Md: Williams & Wilkins; 1999:1862-1874 1999 Self-Assessment Exercise XV. Disorders of the blood and neoplastic disorders [Return to Category List] Questions [Print Directions] Question 2. Answer.

A previously well 6-year-old girl develops icterus, dark urine, and pallor 7 days after an upper respiratory tract infection. Findings include: hemoglobin, 2.9 g/dL; mean corpuscular hemoglobin concentration, 34 g/dL; white blood cell count, 15,700/mm; platelets, 679,000/mm; reticulocyte count, 28%; and total bilirubin, 3.9 mg/dL. Peripheral smear reveals polychromasia and small numbers of spherocytes The MOST likely explanation for these findings is A. B. C. D. E. autoimmune hemolytic anemia glucose-6-phosphate dehydrogenase (G6PD) deficiency hereditary spherocytosis pyruvate kinase deficiency sickle cell beta-thalassemia Answer.

Question 17.

A 10-month-old African girl who recently emigrated from Nigeria presents with tender swelling of her feet. Pertinent laboratory findings include a hemoglobin concentration of 9.7 g/dL and serum bilirubin of 3.5 mg/dL. Radiographs of her feet demonstrate some cortical thinning and irregular areas of radiolucency. Among the following, the MOST likely diagnosis is A. B. C. D. E. juvenile rheumatoid arthritis kwashiorkor malaria multifocal osteomyelitis sickle cell disease Answer.

Question 35.

An 18-month-old girl is being evaluated because her mother thinks her abdomen seems full. Physical examination reveals an abdominal mass. Ultrasonography identifies a solid renal mass. At surgery, a stage I Wilms tumor is found. This childs chance of 4-year survival is CLOSEST to: A. B. C. D. E. 30% 45% 60% 75% 95% Answer.

Question 54.

A 3-year-old girl who has a large abdominal mass has just been diagnosed as having Burkitt lymphoma. She is started on chemotherapy. Of the following, the MOST likely cause of death in the first week of therapy is A. B. C. D. E. anaphylaxis congestive heart failure hyperkalemia lobar pneumonia sepsis Answer.

Question 73.

A 10-year-old boy who has hereditary spherocytosis has developed increasing fatigue and pallor over the past 5 days, and his hemoglobin concentration, which usually ranges between 10 and 11 g/dL, now is 5.9 g/dL. Of the following, the MOST likely etiology of this decrease in hemoglobin is A. B. C. D. E. aplastic crisis folic acid deficiency hyperhemolytic crisis iron-deficiency anemia splenic sequestration Answer.

Question 92.

A 6-year-old girl has had diffuse aching in her arms, legs, and back for more than 2 weeks. Results of laboratory tests include hemoglobin, 9.4 g/dL; white blood cell count, 5,600/mm with no abnormal cells noted on smear; and platelet count, 106,000/mm. Radiographs of long bones reveal osteolytic lesions and radiolucent metaphyseal growth arrest lines. Of the following, the MOST likely cause of these findings is A. B. C. D. E. acute lymphoblastic leukemia aplastic anemia Gaucher disease lead poisoning multifocal osteomyelitis

Question 102. Answer. A 6-year-old boy is brought to your office for evaluation of a palpable purpuric rash on his arms, legs, and trunk. He has a fever of 39.4C (102.9F), appears ill, and complains of headache, arthralgias, and abdominal pain. Of the following, the MOST likely diagnosis is A. B. C. D. E. acute myelogenous leukemia autoimmune thrombocytopenic purpura coxsackievirus infection Henoch-Schnlein purpura meningococcemia

Question 114. Answer. A 4-week-old girl presents for evaluation of a rapidly growing hemangioma on the right eyelid. On physical examination, she barely is able to open her right eye, and you decide that intervention is necessary to prevent amblyopia. Of the following, the most appropriate INITIAL treatment for this child is A. B. C. D. E. administration of oral prednisone freezing with liquid nitrogen injection of intralesional steroids injection of subcutaneous interferon surgical excision of the hemangioma

Question 132. Answer. Of the following, the MOST common initial manifestation of von Willebrand disease in girls is A. B. C. D. E. postoperative bleeding hemarthrosis hematuria intracranial bleeding menorrhagia

Question 170. Answer. Which of the following is MOST likely to be associated with ABO incompatibility? A. B. C. D. E. Anemia in a first-born child Hepatosplenomegaly Hydrops fetalis Kernicterus Severe hemolysis

Question 189. Answer. A 5-month-old girl appears pale. Findings include: liver palpable 2 cm below the right costal margin; spleen palpable 4 cm below the left costal margin; hemoglobin, 5.8 g/dL; mean corpuscular volume (MCV), 53 fL; and white blood cell count, 49,000/mm corrected to 21,000/mm because of a predominance of nucleated erythrocytes in the differential count). Of the following, the MOST likely diagnosis is A. B. C. D. E. erythroleukemia hemoglobin H disease iron-deficiency anemia sideroblastic anemia thalassemia major

Question 207. Answer. Bleeding time is a screening test for evaluating possible bleeding disorders. Of the following, the results of this test are BEST correlated with A. factor VIII antigenic activity

B. C. D. E.

factor VIII coagulant activity fibrin clot formation platelet function vascular stability

Question 227. Answer. An 18-month-old girl has had severe neutropenia for the past year. Her absolute neutrophil count never exceeds 300/mm, and antineutrophil antibodies are absent. She has had two episodes of pneumonia and three episodes of cellulitis. Findings on examination of bone marrow are normal except for an almost total absence of mature polymorphonuclear neutrophils. The MOST appropriate therapeutic step is A. B. administration of broad-spectrum prophylactic antibiotics administration of intravenous gamma globulin

C. bone marrow transplantation from a human leukocyte antigen (HLA)-identical sibling D. E. daily administration of granulocyte colony-stimulating factor (G-CSF) protective isolation

Question 244. Answer. A previously well 9-month-old boy develops increasing pallor and fatigue over 3 weeks. Laboratory findings include: hemoglobin, 2.9 g/dL; mean corpuscular volume (MCV), 72 fL; white blood cell count, 7,400/mm with 40% neutrophils, 54% lymphocytes, and 6% eosinophils; and reticulocyte count, 0.2%. Of the following, the MOST likely explanation for these findings is A. B. C. D. E. aplastic anemia Diamond-Blackfan syndrome iron-deficiency anemia parvovirus infection transient erythroblastopenia of childhood

Answers Critique 2 Preferred Response: A

[View Question]

Acute severe hemolysis due to autoimmune hemolytic anemia typically occurs in otherwise healthy children following an infection of the upper respiratory tract or other organ system. For reasons that remain unclear, these children develop autoantibodies directed against erythrocyte antigens. In most cases, the antibodies cause the destruction of not only the endogenous red cells, but any transfused red cells as well. The majority of children recover, but some quickly develop life-threatening anemia that requires urgent treatment. Other affected children develop a chronic course that can be characterized by remissions, relapses, and a high mortality rate. The signs and symptoms of autoimmune hemolytic anemia are similar to other hemolytic anemias and include pallor, decreased tolerance to exercise, fatigue, and dizziness. The hemolysis results in jaundice as well as dark urine due to hemoglobinuria, increased excretion of bilirubin, or both. Hepatosplenomegaly may be noted, but it generally is not massive. Reticulocytosis is expected, but low reticulocyte counts are not uncommon and do not exclude the diagnosis. The white blood cell and platelet counts usually are normal, but may be elevated in response to the increased erythroid turnover. The peripheral smear usually reveals microspherocytes that can be confused with findings in patients who have hereditary spherocytosis. In patients who have suspected autoimmune hemolytic anemia, the most important laboratory test is the direct antiglobulin or direct Coombs test, which is almost always positive. Other hemolytic anemias also may cause acute hemolysis in previously well children. Microangiopathic processes such as hemolyticuremic syndrome may present with hemolysis and spherocytes, but schistocytes are predominant, and thrombocytopenia usually is evident. Glucose6phosphate dehydrogenase (G6PD) deficiency also causes acute and severe hemolysis. Episodes of acute hemolysis often are associated with intercurrent viral illnesses, but G6PD predominantly affects males, and spherocytosis is not prominent. Hemolysis due to hemoglobinopathies (eg, sickle cell disease) and enzyme deficiencies (eg, pyruvate kinase deficiency) also are not associated with spherocytosis. The key to differentiating all of these disorders from autoimmune hemolytic anemia is the Coombs test, which is usually negative in hereditary spherocytosis, G6PD deficiency, pyruvate kinase deficiency, and sickle cell disease. References: artin PL, Pearson HA. The hemolytic anemias. In: Oski FA, DeAngelis CD, Feigin RD, McMillan JA, Warshaw JB, eds. Principles and Practice of Pediatrics. 2nd ed. Philadelphia, Pa: JB Lippincott Co; 1994:1663-1666 Ware RE, Rosse WF. Autoimmune hemolytic anemia. In: Nathan DG, Orkin SH, eds. Nathan and Oski's Hematology of Infancy and Childhood. 5th ed. Philadelphia, Pa: WB Saunders Co; 1998:499-522 Critique 17 Preferred Response: E

[View Question]

Before the advent of neonatal screening for hemoglobin S, the most common clinical presentation of sickle cell disease in infancy was hand-foot syndrome. This form of vasoocclusive crisis usually occurs in the first 3 years of life. The infant or child cries with pain; refuses to bear weight or use the hands; and develops puffy, tender, warm hands, feet, or both. Such patients may appear acutely ill with fever and can demonstrate impressive leukocytosis. Several conditions should be considered in the differential before establishing the diagnosis of sickle cell disease. Juvenile rheumatoid arthritis also can present with fever, tender swelling of the extremities, and anemia, but hemolysis is much less common. Multifocal osteomyelitis is associated with similar radiologic changes and anemia as sickle cell disease, but the fever usually is greater, and hemolysis is not common. Malaria can cause fever and anemia, but swelling of the digits is not characteristic. Kwashiorkor causes irritability, anemia, and edema, but the edema is not localized to the extremities, and pain is not a prominent symptom. Neonatal screening for sickle cell disease currently is performed in most areas of the United States. This establishes the diagnosis before clinical manifestations are evident and helps to provide access to clinical care, education, and antibiotic prophylaxis before complications develop. This early intervention and education prevents deaths from life threatening illnesses that might be managed less aggressively if the underlying hemoglobinopathy is not recognized. The classic example of such a complication is overwhelming sepsis due to hyposplenia. A major success of neonatal screening for hemoglobins has been the reduction of morbidity and mortality due to overwhelming sepsis. Splenic sequestration is another leading cause of death in patients who have sickle cell disease that can be diagnosed and treated more effectively when parents and clinicians are aware of the underlying disease. Typically, children develop rapid and massive enlargement of the spleen, which traps large volumes of blood and results in hypovolemia. Patients become weak and dyspneic, with a distended abdomen (due to splenomegaly), leftsided abdominal pain, vomiting, and shock. The process can develop within hours, and death may occur even before the child reaches the hospital. Therapy of splenic sequestration and hypovolemia includes emergency transfusion of packed red blood cells to restore intravascular volume and oxygen-carrying capacity. However, administration of intravenous fluids should not be delayed while awaiting red blood cells for transfusion. Teaching parents how to palpate the spleen and stressing the urgency of seeking medical attention for splenomegaly and/or pallor also has reduced mortality substantially. An important condition to be considered in the differential diagnosis of splenic sequestration is aplastic crisis, which can require transfusion, although the more gradual development of anemia generally results in hypervolemia (rather than hypovolemia). Thus, children who have aplastic crisis usually do not tolerate the rapid administration of fluids. References:

Dover GJ, Platt OS. Sickle cell disease. In: Nathan DG, Orkin SH, eds. Nathan and Oski's Hematology of Infancy and Childhood. 5th ed. Philadelphia, Pa: WB Saunders Co; 1998:762810 Graham EA. The changing face of anemia in infancy. Pediatrics in Review. 1994;15:175-183 Lane PA. Sickle cell disease. Pediatr Clin North Am. 1996;43:639-664 Critique 35 Preferred Response: E

[View Question] Therapy of Wilms tumor, which is sensitive to both chemotherapy and radiation therapy, is one of the great success stories of modern pediatric oncology. Before 1950, 75% of children who had this tumor died in childhood; today, the overall survival rate approaches 90%. The 4-year survival rate for children who have stage I disease, as described in the vignette, is greater than 95%. Two thirds of patients present with abdominal swelling or a palpable mass, usually noticed by parents while dressing or bathing the child. A parental concern about an abdominal mass always should be investigated. Less often it is identified on a routine physical examination. The mass usually is hard, smooth, and confined to one side of the abdomen. Abdominal pain, hematuria, and fever are less frequent presenting symptoms. Congenital anomalies, including aniridia, hemihypertrophy, cryptorchidism, and hypospadias, are seen in a small minority of patients. Ultrasonography is the initial diagnostic study of choice and characteristically a solid renal mass is found. Contrastenhanced abdominal computed tomography can define the lesion further. Development and implementation of the National Wilms Tumor Study staging system has been critical in optimizing and individualizing the care of these children. Ongoing studies have continued to improve treatment, often by reducing the intensity of therapy, and, therefore, toxicity without compromising the long-term survival. Staging requires surgical biopsy, hopefully with complete excision, and careful pathologic examination. In stage I disease, the tumor is limited to the kidney and is excised completely. Stage II disease extends beyond the kidney, but can be removed without apparent residual disease. Stage III disease involves residual nonhematogenous tumor confined to the abdomen, and stage IV disease is characterized by hematogenous metastases, most often to the lung, liver, bone, or brain. Stage V disease consists of bilateral disease at the time of diagnosis. Treatment of stages I and II disease is limited to two chemotherapeutic agents (vincristine and dactinomycin), currently administered for 6 months or less for patients who have stage I involvement and 15 months for those who have stage II disease. The 4-year survival rates for patients who have stages I or II disease are at least 95%. Stages III and IV disease are treated more intensively with the addition of doxorubicin and radiation therapy, and the 2-year survival rate approaches 90%. These survival estimates are for children whose tumor histology is classified as favorable, which constitutes nearly 90% of patients.

References: Green DM, Coppes MJ, Breslow NE, et al. Wilms tumor. In: Pizzo PA, Poplack DG, eds. Principles and Practice of Pediatric Oncology. 3rd ed. Philadelphia, Pa: Lippincott-Raven Publishers; 1997:733-760 Petruzzi MJ, Green DM. Wilms' tumor. Pediatr Clin North Am. 1997;44:939-952 Steuber CP, Fernbach DJ. Neoplasms of the kidney or suprarenal area. In: Oski FA, DeAngelis CD, Feigin RD, McMillan JA, Warshaw JB, eds. Principles and Practice of Pediatrics. 2nd ed. Philadelphia, Pa: JB Lippincott Co; 1994:1723-1725 Warner BW. Wilms tumor. Pediatrics in Review. 1996;17:371-372 Critique 54 Preferred Response: C

[View Question] Tumor lysis syndrome is an important cause of morbidity and mortality in children who have certain types of newly diagnosed malignancies. It results from the rapid release of intracellular contents, including potassium, uric acid, and phosphates, from malignant cells. The syndrome develops most often within 1 to 2 days of the initiation of chemotherapy, but it may precede any treatment. Tumor lysis syndrome is prominent in malignancies that have rapid dividing times and high sensitivity to chemotherapy. Burkitt lymphoma, as described in the vignette, is involved most frequently, followed by T-cell leukemia-lymphoma and acute lymphoblastic leukemia. Both the spontaneous and chemotherapy-induced lysis of malignant cells results in the release of uric acid from nucleic acid breakdown, potassium as the principle intracellular cation, and phosphates, which are found in very high concentrations in lymphoblasts. The resulting hyperuricemia and hyperphosphatemia can cause renal failure, which can exacerbate the hyperkalemia and further worsen the hyperuricemia and hyperphosphatemia. Hypocalcemia and tetany can occur as a result of hyperphosphatemia and the precipitation of calcium phosphate salts. Metabolic stabilization of such patients should be completed if possible before beginning chemotherapy, with a primary goal of preserving renal function. Hydration is most critical, and two to four times normal maintenance fluid requirements should be provided to enhance the excretion of uric acid, potassium, and phosphates. Administration of allopurinol acts to inhibit uric acid synthesis. Hydration and alkalinization promote uric acid and phosphate excretion, but excessive alkalinization must be avoided to prevent precipitation of hypocalcemic tetany. Management of these patients includes withholding potassium supplementation and aggressively treating any hyperkalemia that develops. Patients must be observed carefully for the development of renal failure, and some patients require dialysis if this occurs. Although infections (eg, pneumonia, sepsis) may be seen as complications of chemotherapy or its resulting neutropenia, these adverse events are less common than

tumor lysis syndrome and hyperkalemia during the first week of therapy for Burkitt lymphoma. If untreated, hyperkalemia may be fatal. Anaphylaxis is an uncommon occurrence, and congestive heart failure results from cumulative cardiac toxicity from chemotherapy over many months of treatment. References: Kelly KM, Lange B. Oncologic emergencies. Pediatr Clin North Am. 1997;44-809-830 Lange B, O'Neill JA, Goldwein JW, Packer RJ, Ross AJ III. Oncologic emergencies. In: Pizzo PA, Poplack DG, eds. Principles and Practice of Pediatric Oncology. 3rd ed. Philadelphia, Pa: Lippincott-Raven; 1997:1025-1050 Critique 73 Preferred Response: A

[View Question] The most common etiology of severe anemia in children who have hereditary spherocytosis is an aplastic crisis. Although the disorder is congenital, many children remain asymptomatic and undiagnosed because at least 90% usually have mild or no anemia. Even though red blood cell survival often is shortened to only 30 days, enhanced erythropoiesis compensates and allows the maintenance of a stable level of hemoglobin that only rarely is symptomatic. An intercurrent aplastic crisis, such as that described in the vignette, prompts a dramatic worsening in anemia that causes symptoms and often leads to recognition of the underlying hemolytic anemia. Such aplastic crises usually are a result of infection with parvovirus B19. The systemic infection caused by parvovirus may manifest simply as an intercurrent viral illness or may be more typical of erythema infectiosum (eg, fever, chills, lethargy, nausea, vomiting, abdominal pain, respiratory symptoms, myalgias, arthralgias, and the typical "slapped cheek" rash). Parvovirus selectively infects erythroid precursors and inhibits their growth. As a result, reticulocyte counts fall, and bilirubin levels decrease as the red blood cell count drops and the absolute number of red blood cells destroyed daily decreases. White blood cell and platelet precursors also may be affected, although less severely. The subsequent erythroid aplasia generally lasts 10 to 14 days, which causes hemoglobin levels to fall, as seen in the vignette. The result can be a life-threatening anemia. It is important to note that parvovirus infection may pose a serious risk of fetal loss in the first half of the pregnancy term and may result in a loss of the fetus due to severe anemia. Following an aplastic crisis, recovery is heralded by a reticulocytosis. Thus, if the crisis is recognized just as recovery begins, the diagnostic reticulocytopenia may be missed. Hyperhemolytic crises without aplasia also occur in patients who have hereditary spherocytosis, typically in children younger than 6 years of age who have viral syndromes. Common findings include mild and transient increases in bilirubin, splenomegaly, anemia, and an accompanying reticulocytosis. The exacerbation of the anemia usually is not as severe as described in the vignette. Less commonly, folic acid deficiency may exacerbate anemia. Iron-deficiency anemia also may exacerbate the anemia of spherocytosis, but both

iron and folate deficiency result in a gradual fall in hemoglobin. The splenic sequestration crisis typical of sickle cell disease is rarely more than a minor component of the hemolytic crises complicating spherocytosis. References: Bestak M. The aplastic crisis in hereditary spherocytosis (abstract). Pediatrics in Review. 1994;15:212 Gallagher PG, Forget BG, Lux SE. Disorders of the erythrocyte membrane. In: Nathan DG, Orkin SH, eds. Nathan and Oski's Hematology of Infancy and Childhood. 5th ed. Philadelphia, Pa: WB Saunders Co; 1998:544-664 Koch WC, Massey GV. Aplastic crisis. Pediatrics in Review. 1990;12:142-148 Mustafa MM, McClain KL. Diverse hematologic effects of parvovirus B19 infection. Pediatr Clin North Am. 1996;43:809-821 Critique 92 Preferred Response: A

[View Question] The typical child who has acute lymphoblastic leukemia (ALL) presents with bruising or petechiae due to thrombocytopenia and/or pallor and lethargy due to anemia. Classically, lymphoblasts are seen in the peripheral blood. However, the presentation may be varied and at times confusing. Although pancytopenia is classic, the hematologic findings can vary, and some children initially can have relatively normal blood cell counts even when the marrow already has been replaced significantly with leukemic cells. To confuse the issue further, lymphoblasts may not be noted in a smear of peripheral blood. The absence of blasts is common, especially in children who have markedly decreased total white blood cell counts and can lead to an incorrect diagnosis of aplastic anemia. Up to 40% of children who have ALL present with a limp or pain in bones or joints. Bone tenderness frequently is noted, and the long bones are affected most often. It is important to examine the bone marrow when such nonspecific symptoms are present, particularly in association with any hematologic abnormalities. Children who have particularly prominent bone pain are more likely to have nearly normal blood counts, which can cause a delay in diagnosis. Up to 25% of patients have characteristic radiologic changes that are seen most easily in the long bones. As described in the vignette, these include transverse metaphyseal radiolucent bands, osteolytic lesions, and transverse metaphyseal lines of increased density ("growth arrest lines"). Arthralgias also are reported. In patients who have ALL, other conditions such as juvenile rheumatoid arthritis, rheumatic fever, Gaucher disease, infectious mononucleosis, or multifocal osteomyelitis may be misdiagnosed as the cause of these bony findings. Lead poisoning also can cause similar radiologic changes, but not bone pain. References:

Margolin JF, Poplack DG. Acute lymphoblastic leukemia. In: Pizzo PA, Poplack DG, eds. Principles and Practice of Pediatric Oncology. 3rd ed. Philadelphia, Pa: Lippincott-Raven; 1997:409-462 Niemeyer CM, Sallan SE. Acute lymphoblastic leukemia. In: Nathan DG, Orkin SH, eds. Nathan and Oski's Hematology of Infancy and Childhood. 5th ed. Philadelphia, Pa: WB Saunders Co; 1998:1245-1285 Sills RH. Indications for bone marrow examination. Pediatrics in Review. 1995;16:226-228 Critique 102 Preferred Response: E

[View Question] Classically, palpable purpura is associated with leukocytoclastic vasculitis, a dermal inflammatory process in which neutrophils invade the walls of small blood vessels, resulting in vascular destruction and hemorrhage. Meningococcemia is a rapidly fatal disease that may be associated with disseminated intravascular coagulation and widespread bleeding in the skin (purpura fulminans). Palpable purpura and leukocytoclasis (fragmentation and necrosis of leukocytes) are associated with an immune complex-mediated host response that develops in response to this bacterium over several days and is associated with a better prognosis than rapid fulminant disease. Affected patients typically develop high fever, symptoms of meningitis, and small hemorrhagic, necrotic papules in the skin, as described in the patient in the vignette. Early diagnosis and treatment of meningococcemia can decrease the risk of complications and death. In patients who have Henoch-Schnlein purpura (HSP), leukocytoclasis typically involves blood vessels in the skin, gastrointestinal tract, and periarticular soft tissue, resulting in purpuric papules on the buttocks and extensor surfaces of the extremities, colicky abdominal pain, and joint swelling and pain. Vasculitis also can involve the kidneys, central nervous system, and lungs. Affected children usually are afebrile, and symptoms may recur intermittently for several days to weeks. The disorder is self-limiting. Parents of more than 50% of affected children report a gastrointestinal or upper respiratory tract illness during the preceding 4 to 6 weeks, suggesting that HSP is a postinfectious hypersensitivity reaction. A similar reaction has been reported following treatment with a number of drugs, including antibiotics, anticonvulsants, and antihypertensives. Rarely, leukocytoclastic vasculitis can be associated with malignancy, viral infections, or autoimmune thrombocytopenic purpura, but other cutaneous lesions are more characteristic. In patients who have acute myelogenous leukemia, leukemic nodules occasionally appear in the skin. Thrombocytopenia-associated petechiae and ecchymoses are seen in patients who have leukemia and autoimmune thrombocytopenic purpura. Morbilliform and vesicular rashes are typical of coxsackievirus infections. References:

Athreya BH. Vasculitis in children. Curr Opin Rheumatol. 1996;8:477-484 Jennette JC, Falk RC. Small-vessel vasculitis. N Engl J Med. 1997;337:1512-1523 Jessop SJ. Cutaneous leucocytoclastic vasculitis: a clinical and aetiological study. Br J Rheumatol. 1995;34:942-945 Critique 114 Preferred Response: A

[View Question] Most hemangiomas never require therapy, but obstructive amblyopia can result when large tumors block vision for as little as a few weeks in early infancy, such as in the child in the vignette. Treatment with oral prednisone is the safest, most effective option. Prednisone (3 to 5 mg/kg per day) reduces the size of the hemangioma markedly within several days, and medication can be tapered in 3 to 4 months for most children. Although generally well tolerated in infants, prednisone can cause symptoms of colic, sleep disturbances, stress ulcers, and other complications, particularly when high doses are required for long periods of time. Most hemangiomas appear within the first few weeks of life, grow for 3 to 6 months (occasionally up to 12 months), and regress over 5 to 10 years. Although hemangiomas tend to heal with good results when untreated, lesions around the eyes, nares, external ear canal, and visceral lesions (eg, liver, gastrointestinal tract, lungs, and airway) can cause compression of vital structures or bleeding. Rapidly growing, deep dermal and subcutaneous hemangiomas can be associated with disseminated intravascular coagulation and microangiopathic hemolytic anemia (Kasabach-Merritt syndrome). Although oral steroids are the first therapeutic choice for treatment of complicated hemangiomas, daily subcutaneous injection of alpha-interferon is a safe treatment for lifethreatening lesions or for those patients who do not respond to prednisone within several weeks. Intralesional injection of corticosteroids is effective for ocular hemangiomas, but has been associated with serious retinal vascular complications and cutaneous necrosis as well as the usual systemic effects of steroids. Destructive agents (eg, liquid nitrogen, carbon dioxide laser, radiotherapy) can cause unacceptable scarring. Surgical excision should be reserved for small, well-defined lesions or those hemangiomas that fail to respond to medical measures. References: Enjolras O, Riche MC, Merland JJ, Escande JP. Management of alarming hemangiomas in infancy: a review of 25 cases. Pediatrics. 1990;85:491-498 Ezekowitz RA, Mulliken JB, Folkman J. Interferon alfa-2a therapy for life-threatening hemangiomas of infancy. N Engl J Med. 1992;326:1456-1463 Sadan N, Wolach B. Treatment of hemangiomas of infants with high doses of prednisone. J Pediatr. 1996;128:141-146

Wahrman JE, Honig PJ. Hemangiomas. Pediatrics in Review. 1994;15:266-271 Critique 132 Preferred Response: E

[View Question] von Willebrand disease is the most common congenital coagulopathy, affecting 1% or more of the population. It usually is due to an autosomal dominant defect involving the synthesis of von Willebrand protein. This substance has two distinct functions: 1) to carry factor VIII in plasma that allows adequate formation of fibrin and 2) to provide platelets with adherence properties. Diagnosis requires measurement of factor VIII clotting activity as well as functional and antigenic measurements of the von Willebrand proteins, but interpretation of these studies can be complex. The primary clinical manifestations of von Willebrand disease are ecchymoses, epistaxis, menorrhagia, and postoperative bleeding. In adolescent females, menorrhagia is a common presenting symptom. Heavy menstrual bleeding may begin at menarche and be a persistent problem or develop following previously normal menses. There is usually a history of ecchymoses and epistaxis, but because these are common in healthy children following trauma, the underlying coagulopathy often is not recognized at that time. Most symptoms of von Willebrand disease are mild. In evaluating the severity of menorrhagia, it is important to determine the number of pads or tampons used. Heavy bleeding associated with this autosomal dominant disorder may not be recognized by either the mother or daughter because both accept their heavy menorrhagia as normal flow. In a normal menstrual cycle, no more than 10 to 15 pads or tampons are soaked with blood. Actual blood loss usually is 30 to 40 mL; more than 80 mL is excessive. Postoperative bleeding is another common presenting manifestation in girls who have von Willebrand disease. However, because elective surgery is infrequent in children, menorrhagia is more likely to be the initial manifestation of von Willebrand disease. Bleeding related to pregnancy or pregnancy-related surgery occurs less often because factor VIII levels increase during pregnancy. In patients who have von Willebrand disease, hemarthroses, hematuria, and intracranial bleeding are very rare clinical manifestations. In contrast, they are common in patients who have hemophilia. The occurrence of hemophilia in females is extremely rare because this usually is an X-linked disorder. References: Casella JF. Disorders of coagulation. In: Oski FA, DeAngelis CD, Feigin RD, McMillan JA, Warshaw JB, eds. Principles and Practice of Pediatrics. 2nd ed. Philadelphia, Pa: JB Lippincott Co; 1994:1692-1696 Montgomery RR, Gill JC, Scott JP. Hemophilia and von Willebrand disease. In: Nathan DG, Orkin SH, eds. Nathan and Oski's Hematology of Infancy and Childhood. 5th ed. Philadelphia, Pa: WB Saunders Co; 1998:1631-1659

Werner EJ. von Willebrand disease in children and adolescents. Pediatr Clin North Am. 1996;43:683-707 Critique 170 Preferred Response: A

[View Question] ABO hemolytic disease of the newborn is due to maternal anti-A or anti-B immunoglobulin G entering the fetal circulation and destroying fetal erythrocytes. This disease is limited to the offspring of women who have blood group O and only affects babies of blood group A or B. Only mothers who have group O blood type will produce sufficient quantities of anti-A or anti-B antibodies to cause hemolysis. Anti-A and anti-B antibodies are found in all group O individuals. They are "naturally occurring" antibodies from exposure to A and B substances contained in food and bacteria. Therefore, exposure to red blood cells from a prior fetus is not necessary for initial antibody production, and ABO incompatibility occurs as often in first-born infants as in subsequent babies. Indeed, even if the first-born infant is affected, subsequent siblings who are at risk for ABO-incompatibility may not have the disease. This is in clear contrast to Rh disease in which first-born infants rarely are affected, and subsequently born Rh-incompatible siblings tend to develop increasingly severe disease. Maternal-fetal ABO incompatibility exists in 15% of pregnancies, but only 1 in 20 of these infants develop jaundice and only 1 in 150 develop significant hemolysis. Approximately 25% to 50% of ABO-incompatible babies have a weakly positive direct antiglobulin (Coombs) test when cord blood is evaluated. The most common manifestation of both ABO and Rh incompatibility is jaundice within the first 24 hours. However, it is less severe in infants who have ABO incompatibility and often is the only sign of the disease. Hepatosplenomegaly is rare in infants who have ABO incompatibility. Anemia occurs, but the hemoglobin level usually does not drop below 10 g/dL. Serious complications, such as kernicterus and hydrops fetalis, also are rare. Treatment of infants who have ABO incompatibility primarily involves stabilizing the hyperbilirubinemia with phototherapy. It is extremely unusual for exchange transfusion to be required in these infants. One benefit of ABO incompatibility in a mother who is Rh-negative is that it helps protect the fetus from concomitant Rh incompatibility. It is presumed that the maternal erythrocytes that are Rh-positive are destroyed by the anti-A or anti-B antibodies before isoimmunization can occur. References: Bowman JM. Immune hemolytic disease. In: Nathan DG, Orkin SH, eds. Nathan and Oski's Hematology of Infancy and Childhood. 5th ed. Philadelphia, Pa: WB Saunders Co; 1998:53-78 Foerster J. Alloimmune hemolytic anemias. In: Lee GR, Bithell TC, Foerster J, Athens JW, Lukens JN, eds. Wintrobe's Clinical Hematology. 9th ed. Philadelphia, Pa: Lea & Febiger; 1993:1146-1169 Critique 189 Preferred Response: E

[View Question] The initial findings in most patients who have thalassemia major is a severe hypochromic, microcytic anemia. The disorder, also known as homozygous beta-thalassemia or Cooley anemia, is most common in people who are of Italian or Greek descent. The complete or almost complete absence of the beta-globin subunit of hemoglobin A in these individuals results in absent or severely reduced synthesis of hemoglobin A. This anemia does not develop in affected neonates because of the predominance of hemoglobin F, which does not contain a beta-globin subunit. Following the decrease in hemoglobin F production after birth, hemoglobin levels fall and severe anemia usually becomes clinically evident between 6 months and 2 years of age. Children who have thalassemia major typically present with pallor, irritability, growth retardation, abdominal swelling due to hepatosplenomegaly, and jaundice. Characteristic laboratory findings in children who have thalassemia major include hypochromia and microcytosis; hemoglobin concentrations that may be as low as 3 to 4 g/dL; mean corpuscular volume, 50 to 60 fL; and mean corpuscular hemoglobin, 12 to 18 pg per cell. Nucleated erythrocytes are abundant and may result in extreme spurious elevations in white blood cell counts when measured using electronic counters because any nucleated cells (including nucleated erythrocytes) will be counted as white blood cells by these counters. The true white blood cell count also may be elevated because of the markedly increased stimulus for erythropoiesis. Accordingly, findings of severe hypochromic anemia with a preponderance of nucleated erythrocytes, as described in the infant in the vignette, almost always is due to thalassemia major. Hemoglobin H disease, a form of alpha thalassemia, has a clinically similar, but milder presentation than thalassemia major. Some affected infants may be diagnosed in early infancy because the defect of alpha-globin chain synthesis affects the production of both hemoglobin F and hemoglobin A. Hemoglobin H disease (and other forms of alpha thalassemia) are most common in Asian populations. Iron-deficiency anemia is not associated with hepatosplenomegaly or a prominence of nucleated erythrocytes in the peripheral blood and rarely is associated with hemoglobin levels as low as 5.8 g/dL. Sideroblastic anemias are microcytic, but they are extremely rare and usually are not associated with large numbers of nucleated erythrocytes in the blood. Erythroleukemia is an extremely rare condition in which erythroblasts constitute more than 50% of the bone marrow. Children who have thalassemia major require chronic transfusions. Maintaining their hemoglobin levels at a minimum of 9 to 10 g/dL decreases clinical symptoms and results in nearly normal growth and development. However, before the first transfusion is administered, a complete genotype of the red blood cells should be obtained because red blood alloimmunization is more likely when a child is receiving multiple transfusions. The primary complication of transfusion therapy is the inevitable development of hemosiderosis, which, if untreated, ultimately is fatal. Unfortunately, the only currently available treatment is deferoxamine, an iron chelator that must be administered as a daily subcutaneous

infusion over 10 to 12 hours. Chelation is expensive and difficult to administer, and compliance, especially in adolescents, is poor. Several oral chelators are under investigation. If a sibling who is an identical human lymphocyte antigen (HLA) match is available, bone marrow transplantation can cure the majority of patients who have thalassemia. However, there is considerable morbidity and mortality associated with the procedure, and the decision to undertake a transplant remains complex. Unfortunately, no identical donor is available in most cases, and bone marrow transplant is not even an option. References: Martin PL, Pearson HA. The hemoglobinopathies and thalassemias. In: Oski FA, DeAngelis CD, Feigin RD, McMillan JA, Warshaw JB, eds. Principles and Practice of Pediatrics. 2nd ed. Philadelphia, Pa: JB Lippincott Co; 1994:1660-1663 Orkin SH, Nathan DG. The thalassemias. In: Nathan DG, Orkin SH, eds. Nathan and Oski's Hematology of Infancy and Childhood. 5th ed. Philadelphia, Pa: WB Saunders Co; 1998:811886 Critique 207 Preferred Response: D

[View Question] Historically, the bleeding time has been used to evaluate platelet function qualitatively. The test involves creating a superficial cutaneous wound under increased venous pressure and observing the length of time until spontaneous hemostasis occurs. Hemostasis in such superficial wounds is dependent upon a normal platelet count, normal platelet function, and normal vascular stability, but not upon fibrin formation. The bleeding time, therefore, is prolonged in the presence of thrombocytopenia, platelet dysfunction, and in some patients who have vascular abnormalities. In fact, a bleeding time generally is not performed in a patient who has known thrombocytopenia. Isolated disorders of fibrin formation usually will not prolong the bleeding time. The bleeding time has limitations that are particularly significant in children. The results of bleeding times must be interpreted with great care, especially in young children. Standardization of the depth and size of the wound is critical in performing the test. In a struggling child, creating such a standard wound child while maintaining a constant level of increased venous pressure with a sphygmomanometer is very difficult. If the wound is too deep or the venous pressure too high, the bleeding time will be falsely prolonged; if the wound is too shallow or the pressure too low, a false-negative result occurs. In addition, individuals who have vascular abnormalities without platelet defects (eg, generalized vasculitis, scurvy) also can have prolonged bleeding times. The bleeding time is extremely sensitive to the recent ingestion of a variety of drugs that can impair platelet function. Aspirin has dramatic effects that last for days, although this agent rarely is used now in children. Other nonsteroidal anti-inflammatory drugs will have a similar, although more short-lived effect. Because of all these limitations, a single prolonged bleeding time should be confirmed by repeating the test 1 to 2 weeks later (if possible after discontinuing any

antiplatelet agents for at least 10 days). An additional problem is that small scars may occur at the test site, and parents should be warned about this possibility. Platelet function abnormalities are suspected in children who have suspicious clinical histories for a hematologic abnormality yet whose complete blood count, platelet count, peripheral blood smear, partial thromboplastin time, prothrombin time, and factor VIII activities are normal. Although bleeding time can be performed in such patients, more specific platelet function tests (eg, platelet aggregation studies) are more reliable. References: Casella JF. Disorders of coagulation. In: Oski FA, DeAngelis CD, Feigin RD, McMillan JA, Warshaw JB, eds. Principles and Practice of Pediatrics. 2nd ed. Philadelphia, Pa: JB Lippincott Co; 1994:1685-1687 Lusher JM. Approach to the bleeding patient. In: Nathan DG, Orkin SH, eds. Nathan and Oski's Hematology of Infancy and Childhood. 5th ed. Philadelphia, Pa: WB Saunders Co; 1998:1574-1584 Critique 227 Preferred Response: D

[View Question] The isolation, identification, and synthesis of hematologic growth factors has provided tremendous new tools for the treatment of many disorders. Granulocyte colony-stimulating factor (G-CSF) is one of these regulatory molecules, generally known as cytokines. It is an effective treatment for patients who have severe forms of neutropenia that predispose them to serious or recurrent bacterial infections. Administration of G-CSF may elevate the neutrophil count enough to decrease the incidence of or prevent infections. Unfortunately, children who have chronic neutropenia require long-term administration of G-CSF by subcutaneous injections. Although the short-term toxicity is minimal, questions remain about potential long-term toxicities of an agent that has been used only for a few years. There is particular concern about whether there will be any increased risk of leukemias in children who have congenital neutropenia and have received G-CSF. Unfortunately, it will not be known if these risks will be related to the underlying primary disease or to the longterm use of the G-CSF cytokine. For many of these children who have chronic neutropenia, there are few other therapeutic options. Management of each febrile episode includes obtaining a blood count and blood cultures and, in the presence of severely neutropenia, treating with broadspectrum intravenous antibiotics. Protective isolation is not helpful, and administration of prophylactic antibiotics is of very limited usefulness. Bone marrow transplantation may be undertaken if an HLA-identical sibling donor is available, but this approach would be considered only if there is a poor response to G-CSF or if the child develops leukemia. Administration of intravenous gamma globulin can be useful in patients who have autoimmune neutropenia, but G-CSF also may be helpful in such patients. Autoimmune neutropenia usually is suspected on the basis of normal or increased neutrophilic precursors in the marrow; the presence of antineutrophil antibodies establishes the diagnosis.

G-CSF also has been used to reduce the severity and duration of neutropenia in children receiving chemotherapy. Although the duration of neutropenia is shortened and morbidity probably is reduced, it has not been documented that G-CSF reduces mortality. The efficacy of G-CSF in these children currently is being evaluated. Several other cytokines also are available for clinical use or are under investigation, including erythropoietin, granulocyte-macrophage colony-stimulating factor (GM-CSF), several interleukins, and thrombopoietin. References: Boxer LA, Blackwood RA. Leukocyte disorders: quantative and qualitative disorders of the neutrophil, Part 1. Pediatrics in Review. 1996;17:19-28 Kim SK, Demetri GD. Chemotherapy and neutropenia. Hematol Oncol Clin North Am. 1996;10:377-395 Sieff CA, Nathan DG, Clark SC. The anatomy and physiology of hematopoiesis. In: Nathan DG, Orkin SH, eds. Nathan and Oski's Hematology of Infancy and Childhood. 5th ed. Philadelphia, Pa: WB Saunders Co; 1998:161-236 Critique 244 Preferred Response: E

[View Question] Transient erythroblastopenia (TEN) of childhood is characterized by the development of severe anemia and reticulocytopenia in a previously healthy child. The peak incidence is at 2 years of age; more than 80% of patients are older than 12 months. A history of a viral illness several weeks before is common, but the pathophysiology of this disorder remains poorly understood. The anemia is normochromic and normocytic, and mean hemoglobin levels usually are less than 6 g/dL. Reticulocyte counts are below 1%, except in children who already have begun to recover. Platelet and white blood cell counts generally are normal, although absolute neutrophil counts less than 1,000 cells/mm3 are noted in approximately 20% of patients. Bone marrow specimens usually demonstrate decreased erythroid precursors. The natural history of TEN is spontaneous recovery, which usually occurs within 1 to 2 months following diagnosis. Indeed, 5% to 10% of patients already are recovering at the time of diagnosis. Because patients in the recovery phase may have reticulocytosis rather than reticulocytopenia, they may be thought to have a hemolytic anemia. However, they do not have hyperbilirubinemia, and their hemoglobin levels are rising rather than dropping along with the reticulocytosis. Current therapy recommendations for patients who have TEN include careful observation and administration of transfusions only if cardiovascular compromise from severe anemia begins to develop. In patients who require a transfusion, a second one rarely is required. Drugs such as prednisone, anabolic steroids, or immunosuppressive agents should not be administered.

The primary condition to be considered in the differential diagnosis is DiamondBlackfan syndrome, which is much rarer than TEN and usually presents at 2 months of age rather than 2 years. Fewer than 15% of patients are older than 1 year of age. Approximately 20% to 25% of patients also have congenital malformations and 80% demonstrate macrocytosis. All patients who have Diamond-Blackfan syndrome have elevated levels of fetal hemoglobin and increases in the fetal red blood cell "i" antigen compared with only 25% and 20%, respectively, of patients who have TEN. Intercurrent viral or bacterial illnesses are a common cause of normocytic, normochromic anemia in children, but the anemia rarely is severe. Aplastic anemia can be differentiated easily from TEN by the presence of pancytopenia rather than an isolated anemia. Parvovirus infection usually causes severe anemia only in children who have underlying hemolytic anemias or immunodeficiencies. Iron-deficiency anemia is common in children younger than 1 year of age, but it almost always is associated with microcytosis rather than a normochromic anemia, especially when the anemia is severe. References: Alter BP, Young NS. The bone marrow failure syndromes. In: Nathan DG, Orkin SH, eds. Nathan and Oski's Hematology of Infancy and Childhood. 5th ed. Philadelphia, Pa: WB Saunders Co; 1998:237-335 Martin PL, Pearson HA. The hypoplastic and aplastic anemias. In: Oski FA, DeAngelis CD, Feigin RD, McMillan JA, Warshaw JB, eds. Principles and Practice of Pediatrics. 2nd ed. Philadelphia, Pa: JB Lippincott Co; 1994:1667-1670 1998 Self-Assessment Exercise XV. Disorders of the blood and neoplastic disorders [Return to Category List] Questions [Print Directions] Question 5. Answer.

Among the following, the factor that would have the MOST adverse effect on the long-term prognosis of an 8-year-old boy who has a specific learning disability is a(n) A. B. C. D. E. adjustment reaction to his parents' divorce associated attention deficit hyperactivity disorder intelligence quotient of 85 myelomeningocele at L5-S1 unilateral hearing loss of 30 dB Answer.

Question 8.

A firm abdominal mass is discovered in a 3-month-old boy. Radiography reveals a calcified suprarenal mass pushing the left kidney inferiorly with no bony involvement. Urinary catecholamines are elevated. Liver involvement and scattered neuroblastoma cells in the bone marrow are identified. The N-myc oncogene from the tumor is not amplified. After complete resection of the mass, the most appropriate INITIAL course of action is A. B. C. D. E. chemotherapy alone close observation intensive chemotherapy rescued by autologous bone marrow transplantation local radiotherapy to the kidneys plus chemotherapy low-dose total body irradiation Answer.

Question 48.

Which of the following findings is MOST likely to be a sign of neutropenia? A. B. C. D. E. disseminated herpes zoster mucosal ulcerations scabies infestation subcutaneous nodules urticaria

Question 128. Answer. A healthy 2-day-old boy born at term undergoes circumcision prior to discharge from the hospital. Bleeding was noted at the site of circumcision 10 hours after the procedure and has increased steadily over the past 4 hours. Findings on physical examination are unremarkable except for bleeding along 2 to 3 mm of the surgical site; there are no petechiae or purpura. Of the following, the MOST likely cause of the bleeding is A. B. C. D. E. disseminated intravascular coagulation factor VIII deficiency hemophilia immune thrombocytopenic purpura neonatal alloimmune thrombocytopenia von Willebrand disease

Question 162. Answer.

A 7-year-old girl, diagnosed at 4 years of age as having acute lymphoblastic leukemia, successfully completed chemotherapy 1 year ago. She now develops petechiae, purpura, lymphadenopathy, and hepatosplenomegaly. Laboratory findings include: platelet count, 12,000/mm; hemoglobin, 8.0 gm/dL; and white blood count, 13,000/mm. Of the following, the MOST likely explanation for these findings is A. B. C. D. E. acute nonlymphoblastic leukemia as a second malignancy disseminated varicella drug-induced immune thrombocytopenic purpura late-onset aplastic anemia due to chemotherapy viral-induced immune thrombocytopenic purpura

Question 207. Answer. A 10-week-old boy has an inguinal hernia that was identified at the time of a health supervision visit. The results of laboratory studies performed prior to surgery include: hemoglobin, 9.7 g/dL; hematocrit, 29.2%; mean corpuscular volume, 88 fL; white blood cell count, 12,200/mm; platelet count, 349,000/mm. The most appropriate INITIAL approach to management is to perform A. B. C. D. E. a bone marrow examination a diagnostic trial of oral iron therapy a serum ferritin level and transferrin saturation study blood typing and a direct Coombs test no laboratory studies

Question 241. Answer. Laboratory studies performed on a 2-year-old boy undergoing surgery to correct a club foot deformity reveal normal results of a complete blood count except for a platelet count of 8,000/mm and normal prothrombin and partial thromboplastin times. He is otherwise healthy and is not receiving any medications. Physical examination of this boy is MOST likely to reveal A. B. C. D. deep hematomas within muscle dried blood in the perianal region hemarthrosis with a tender, swollen joint petechiae and purpura

E.

raised purpuric lesions

Answers Critique 5 Preferred Response: B

[View Question] Approximately 6% to 8% of all school-age children are identified as having learning disabilities and receive special education services. The long-term effects of learning disabilities vary among individuals; they may influence a specific area such as academics, behavior, or neurologic function or overlap several areas. Attention deficit hyperactivity disorder (ADHD) has been reported in approximately 40% of children who have an identified learning disability. Indeed, children who have learning disabilities are seven times more likely to have ADHD than the general population. The ability to compensate for this associated condition is a crucial determinant of the outcome of learning abilities as children reach adulthood. A unilateral hearing deficit of 30 dB is classified as a mild hearing loss, such as occurs with serous otitis media, tympanosclerosis, or membrane perforation. Such a deficit should result only in a mild auditory dysfunction in language learning. An intelligence quotient of 85 is within the range of normal (ie, 70 to 130) and would have no significant additive effect on the long-term prognosis of learning disability. Children who have meningomyelocele in the lumbosacral region have an increased incidence of learning disabilities compared with the general population, but in the absence of shunt malfunction or infection, this should have little additional impact. Although the initial academic difficulty defines this diagnosis, other associated deficits, such as low self-esteem, hyperactivity, poor social interaction, and family dysfunction (eg, divorce), may have a significant impact on the overall outcome and may change over time. It often is reported that children "outgrow" their learning problems, which has led some authorities to suggest that fewer services will be required as children mature. However, adolescence may be a time when some children, who previously have compensated for their disabilities, become overwhelmed by the demands of time management, organizational and study skills, multiple assignments, and long-term projects. Such adolescents may require an increase in individualized educational services. References: Capin DM. Developmental learning disorders: clues to their diagnosis and management.03624 Pediatrics in Review. 1996;17:284-290 DuPaul GJ, Stoner G. ADHD and learning difficulties: what is the connection? In: ADHD in the Schools: Assessment and Intervention Strategies. New York, NY: The Guilford Press; 1994:6295 Shapiro BK, Gallico RP. Learning disabilities. Pediatr Clin North Am. 1993;40:491-505

Critique 8

Preferred Response: B

[View Question] Neuroblastoma comprises only 7% of childhood malignancies, but it is the most common congenital cancer. A primary abdominal location and calcification of the mass, as described in the vignette, are common. The finding of elevated urinary catecholamines is almost pathognomonic for neuroblastoma. Neuroblastoma is unique in the variability of its prognosis. The strongest predictive factors include the child's age; the anatomic extent of disease (or "stage"); and the specific characteristics of the tumor, including histology, tumor cell ploidy, and amplification of the MYCN (N-myc) gene. Children younger than 1 year of age who have stage 1, 2, or 4S disease; favorable histology; hyperdiploid karotype; and lack of amplification of the MYCN gene have a 3-year survival rate of approximately 95%. In contrast, children older than 1 year of age who have stage 3 or 4 disease, unfavorable histology, near-diploid tumor cell ploidy, and amplification of the MYCN gene have a 3-year survival rate of approximately 5%. Several staging classifications have been used for neuroblastoma. The newer International Neuroblastoma Staging System, which is based on clinical, radiologic, and surgical evaluation, currently is recommended. Briefly, stage 1 disease is localized to the area of origin and is removed completely with no apparent residual disease. Stage 2A is an incompletely resected unilateral tumor without ipsilateral lymph node involvement, and stage 2B disease involves ipsilateral (but not contralateral) lymph nodes. Stage 3 disease crosses the midline with or without node involvement. Stage 4 disease has disseminated distantly to lymph nodes, bone, bone marrow, liver, or other organs (except as defined in stage 4S). Stage 4S describes a localized primary tumor, as defined in stage 1 or 2, that includes metastatic disease limited to liver or skin or involvement of less than 10% of the bone marrow. Infants who have stage 4S neuroblastoma, such as the infant in the vignette, are of particular interest. Despite the presence of metastatic disease, infants older than 6 weeks who have disease at this stage have a 3-year survival rate of more than 80%. Although their management remains to be standardized, intervention usually is limited to those infants who develop respiratory compromise from massive hepatomegaly. Even in such infants, however, treatment generally is limited to minimal chemotherapy or local irradiation unless there is obvious progression of disease. Infants who have no life-threatening complications are observed without therapy; in most, disease will begin to reverse and ultimately enter remission spontaneously. Accordingly, close observation, rather than more aggressive interventions, is the most appropriate management for the infant in the vignette. References: Brodeur GM, Castleberry RP. Neuroblastoma. In: Pizzo PA, Poplack DG, eds. Principles and Practice of Pediatric Oncology. 3rd ed. Philadelphia, Pa: Lippincott-Raven Publishers; 1997:761-797

Dreyer ZE, Fernbach DJ. Neuroblastoma. In: Oski FA, DeAngelis CD, Feigin RD, McMillan JA, Warshaw JB, eds. Principles and Practice of Pediatrics. 2nd ed. Philadelphia, Pa: JB Lippincott Co; 1994:1726-1729 Critique 48 Preferred Response: B

[View Question] Neutropenia in children usually is a transient phenomenon resulting from a viral illness. Rarely, serious bacterial infection or other complications of neutropenia occur. However, children who have more severe, and often chronic, neutropenia can develop potentially lifethreatening complications. Patients who have neutropenia may present with severe bacterial infections, but they also may present with mucositis alone, which can progress to necrosis of the mucous membranes. The mucositis generally consists of single or multiple ulcers and most commonly involves the oral mucosa. Patients who have isolated neutropenia do not have an increased risk of herpes zoster. It usually is associated with immunosuppression, and it occurs most frequently in patients who have received chemotherapy or in those who have congenital or acquired immunodeficiencies (eg, human immunodeficiency virus infection). Scabies infections are not associated with neutropenia. Subcutaneous nodules and urticaria also are not seen commonly in association with neutropenia unless there is a more complex underlying etiology, such as a collagen vascular disorder. References: Baehner RL. Neutropenia. In: Nelson WE, Behrman RE, Kliegman RM, Arvin AM, eds. Nelson Textbook of Pediatrics. 15th ed. Philadelphia, Pa: WB Saunders Co; 1996:587-591 Bernini JC. Diagnosis and management of chronic neutropenia during childhood. Pediatr Clin North Am. 1996;43:773-792 Critique 128 Preferred Response: B

[View Question] Excessive bleeding following circumcision may be an early warning sign of a congenital coagulopathy. The disorder of greatest concern is hemophilia due to either factor VIII or factor IX deficiency. Although most infants who have hemophilia do not bleed excessively following circumcision, this may be the only clinical sign of the disease until the infant is several months old. The bleeding often is delayed for hours or days, and there may be no other manifestations. A careful family history of bleeding disorders should be obtained. Because the gene is X-linked, particular attention should be paid to the medical history of males related to the infant through the maternal line. However, the family history often is negative in a child who has hemophilia because new spontaneous mutations account for approximately one third of all cases.

The appropriate laboratory studies to obtain immediately in the infant described in the vignette are a complete blood count, platelet count, partial thromboplastin time (PTT), and prothrombin time (PT), and factor VIII and factor IX levels. Although the PTT is relatively prolonged in patients who have severe hemophilia, the significantly prolonged normal values in neonates and the relative insensitivity of the PTT to mild or moderate congenital factor deficiencies requires measurement of specific factor assays. If results of these laboratory studies are normal, additional evaluation is indicated if the bleeding is severe or the family history is suggestive. Von Willebrand disease rarely presents with bleeding in the neonatal period. Disseminated intravascular coagulation would not present without other evidence of clinical bleeding and usually does not occur in an otherwise healthy infant. Platelet disorders, such as immune thrombocytopenic purpura and neonatal alloimmune thrombocytopenia, rarely present with bleeding limited to the site of circumcision, and typically they are associated with immediate rather than delayed postoperative bleeding. Immune thrombocytopenic purpura is rare in the neonate. References: Corrigan JJ. Factor VIII deficiency. In: Nelson WE, Behrman RE, Kliegman RM, Arvin AM, eds. Nelson Textbook of Pediatrics. 15th ed. Philadelphia, Pa: WB Saunders Co; 1996:1424-1427 Montgomery RR, Scott JP. Hemostasis: diseases of the fluid phase. In: Nathan DG, Oski FA, eds. Hematology of Infancy and Childhood. 4th ed. Philadelphia, Pa: WB Saunders Co: 1993:1615-1618 Critique 162 Preferred Response: A

[View Question] Approximately 1 in 1,000 young adults are survivors of childhood cancer. Therefore, the long-term effects of childhood cancer and its treatment are of increasing importance to the pediatrician. The long-term effect of greatest concern is the development of new malignancies, either as a consequence of the treatment, an inherent disposition to develop cancers, or both. The rate of second malignant neoplasms in the first 20 years following the initial diagnosis of childhood cancer is 3% to 12%, which is 10 to 20 times that of the general population. The risk for any individual patient is highly dependent upon the original diagnosis, patient age, the specific therapy, and the presence of associated genetic conditions. Patients at greatest risk of second malignant neoplasms include those who have Hodgkin disease, ovarian cancer, retinoblastoma, or the genetic form of Wilms tumor; those treated with radiation therapy or alkylating agents; and those who have neurofibromatosis and immunodeficiency syndromes. The risk of second malignancies is lower in children treated for acute lymphoblastic leukemia, but those receiving intensive treatment regimens are at substantial risk. Accordingly, the most likely diagnosis for the child in the vignette,

who has anemia, thrombocytopenia, hepatosplenomegaly, and lymphadenopathy, is acute nonlymphoblastic leukemia as a second malignancy. Disseminated varicella is unlikely without the characteristic rash. Both viral and drug-induced immune thrombocytopenic purpura (ITP) are associated with normal hemoglobin levels. In addition, neither ITP nor aplastic anemia is associated with hepatosplenomegaly. Survivors of childhood cancer remain at substantial risk for other long-term complications. Gonadal dysfunction usually is the result of therapy. Testicular radiation causes dose-dependent organ damage that often is irreversible if high doses were administered. Chemotherapy, particularly with the alkylating agents and procarbazine, can cause irreversible azoospermia and have adverse effects on Leydig cell function, causing low testosterone levels. The prepubertal testes appear to be more resistant to these toxicities. Gonadal dysfunction also can occur in females. Ovarian radiation can result in irreversible gonadal failure. The ovary is more resistant to the toxicity of chemotherapy than the testes, but its function can be impaired, particularly by alkylating agents. Intrapartum chemotherapy is of great concern. Another issue is the potential risks to children born to cancer survivors; little is known about this risk, and long-term follow-up is essential. Other long-term complications of cancer and its treatent include hypothyroidism and impaired growth. Although catch-up growth after treatment is common, cranial irradiation especially can result in a reduced adult height. Radiation also causes musculoskeletal changes, such as scoliosis or other deformities, depending on the dose of radiation and the areas treated. Neuropsychologic and neurologic function also can be impaired, most often related to the use of cranial irradiation. Chronic, and often delayed, cardiomyopathy is a great concern, particularly with the use of the anthracyclines (eg, doxorubicin, daunomycin). Pulmonary fibrosis can result from radiation or from chemotherapeutic agents such as bleomycin. Long-term follow-up of cancer survivors is essential and requires continued close cooperation between the primary pediatrician and the pediatric oncologist. References: Blatt J, Copeland DR, Bleyer WA. Late effects of childhood cancer and its treatment. In: Pizzo PA, Poplack DG, eds. Principles and Practice of Pediatric Oncology. 3rd ed. Philadelphia, Pa: Lippincott-Raven Publishers; 1997:1303-1329 Meistrich ML, Vassilopoulou-Sellin RV, Lipshultz LI. Gonadal dysfunction. In: DeVita VT Jr, Hellman S, Rosenberg SA, eds. Cancer: Principles & Practice of Oncology. 5th ed. Philadelphia, Pa: Lippincott-Raven Publishers; 1997:2758-2772 Neglia JP, Meadows AT, Robison LL, et al. Second neoplasms after acute lymphoblastic leukemia in childhood. N Engl J Med. 1991;325:1330-1336

Schwartz CL, Hobbie WL, Constine LS, Ruccione KS. Survivors of Childhood Cancer. St Louis, Mo: Mosby-Year Book, Inc; 1994 Critique 207 Preferred Response: E

[View Question] Hemoglobin levels vary most in the first few months of life, depending primarily on both chronologic age and gestational age at birth. When evaluating the hemoglobin of infants in the first 6 months of life, it is particularly important to compare the results with standards adjusted for age and prematurity. Without such comparisons, many infants will be misdiagnosed as having anemia. Unfortunately, use of the term "physiologic anemia" to describe the comparatively low but normal values of hemoglobin at ages 2 to 3 months adds to the confusion. Relatively high levels of hemoglobin are necessary in the fetus to provide increased oxygen-carrying capacity in the presence of intrauterine hypoxemia. The onset of pulmonary gas exchange at birth dramatically increases arterial oxygen saturation in the neonate, making higher levels of hemoglobin unnecessary. Hemoglobin concentrations fall steadily from birth until they reach a nadir at approximately 2 months of age. At that point, erythropoiesis increases and hemoglobin levels gradually rise. The lower limit (2 standard deviations below the mean) of normal hemoglobin levels at birth from cord blood samples generally is considered to be 13.6 g/dL. No real decrease occurs in the term infant between the first and third weeks of life, but the hemoglobin level then falls steadily so that at 2 months of age the lower limit of normal for a term infant is 9.0 g/dL. Thus, the infant in the vignette should be considered as having a normal level of hemoglobin. Accordingly, no laboratory studies are indicated. Preterm infants weighing 1,501 to 2,000 g have a lower limit of normal hemoglobin of 8.0 g/dL at 2 months, while those weighing 1,000 to 1,500 g can have levels as low as 7.1 g/dL and still remain in the normal range. The fall in hemoglobin in the first 2 to 3 months of life is not related to iron deficiency; the fetus avidly acquires iron from the mother, even if the mother is irondeficient. Iron stores in healthy neonates are so adequate that iron deficiency is rare before the age of 4 to 5 months. Iron deficiency in the neonate is limited to rare instances of chronic in utero blood loss. As hemoglobin levels normally fall in the first 2 to 3 months of life, iron is released from degraded hemoglobin and stored for later use. Assuming that iron stores are normal at the time of birth and that there is no ongoing blood loss, iron stores should not be exhausted until an infant has more than doubled his or her birthweight. Preterm infants, who usually double their birthweights at an earlier age, tend to develop iron deficiency earlier. Routine iron supplementation of preterm infants generally prevents this. References:

Buchanan GR. Hematopoietic diseases. In: Oski FA, DeAngelis CD, Feigin RD, McMillan JA, Warshaw JB, eds. Principles and Practice of Pediatrics. 2nd ed. Philadelphia, Pa: JB Lippincott Co; 1994:476-486 Mentzer WC. Blood and blood-forming tissues. In: Rudolph AM, Hoffman JIE, Rudolph CD, eds. Rudolph's Pediatrics. 20th ed. Stamford, Conn: Appleton & Lange; 1996:1167-1172 Schwartz E. Physiologic anemia of infancy. In: Nelson WE, Behrman RE, Kliegman RM, Arvin AM, eds. Nelson Textbook of Pediatrics. 15th ed. Philadelphia, Pa: WB Saunders Co; 1996:1383-1384 Critique 241 Preferred Response: D

[View Question] Bleeding symptoms can result from disorders of platelets, of fibrin formation (coagulation disorders), or both. The child in the vignette presents with severe, isolated thrombocytopenia discovered prior to elective surgery. Hemoglobin concentration, white blood cell count, and absolute neutrophil count are normal. The possibility of an associated coagulation abnormality is unlikely because of the normal prothrombin and partial thromboplastin times. Children who have platelet disorders, whether quantitative or qualitative, usually present with petechiae and purpura. Gastrointestinal bleeding, including perianal bleeding, can occur, but is uncommon. The otherwise healthy child who presents with isolated thrombocytopenia usually has immune thrombocytopenic purpura (ITP). Common conditions that should be considered in the differential diagnosis of ITP include congenital thrombocytopenia (there should be a prior history of bleeding), viral-induced suppression (which rarely causes the degree of isolated thrombocytopenia demonstrated in the child in the vignette), and microangiopathic hemolytic anemias (which usually are associated with severe acute illness and hemolytic anemia). Malignant disorders or aplastic anemia rarely present with isolated thrombocytopenia. There are no laboratory studies that are diagnostic of ITP, so it remains a clinical diagnosis. ITP is generally an acute, self-limited condition, presumably triggered by an intercurrent viral illness. Rarely it may be precipitated by drugs (eg, quinidine, sulfonamides). The symptoms of drug-induced ITP may be quite severe, and it is essential to discontinue the drug. Chronic ITP develops in about 10% to 20% of children who have ITP. In most cases, no specific etiology is found, although collagen vascular disorders, immune deficiencies, and human immunodeficiency virus infection must be excluded. Palpable petechiae or purpura can cause a vasculitis (and raised petechiae) that also can be associated with thrombocytopenia. However, this would be unlikely in an afebrile child who has no other signs of systemic illness. In most cases, there is progressive deterioration with evidence of purpura and in some cases, fulminant meningococcemia may

result in death. Deep hematomas in muscles or hemarthroses in joints rarely are caused by a platelet disorder, but are typical of coagulation abnormalities, such as hemophilia. References: Beardsley DS. Platelet abnormalities in infancy and childhood. In: Nathan DG, Oski FA, eds. Hematology of Infancy and Childhood. 4th ed. Philadelphia, Pa: WB Saunders Co: 1993:15611593 Corrigan JJ. Platelet and blood vessel disorders. In: Nelson WE, Behrman RE, Kliegman RM, Arvin AM, eds. Nelson Textbook of Pediatrics. 15th ed. Philadelphia, Pa: WB Saunders Co; 1996:1431-1435 Medeiros D, Buchanan GR. Current controversies in the management of idiopathic thrombocytopenic purpura during childhood. Pediatr Clin North Am. 1996;43:757-772

Вам также может понравиться